You are on page 1of 65

IDMP TEST SERIES-2023

High Yielding PRELIMS TEST SERIES

Test-26
Economy
Topic Covered:
❖ Agriculture
❖ Infrastructure and manufacturing
❖ Industry
❖ Services sector
❖ Current Affairs of September 2022


IDMP-2023
Contact us :info@onlyias.com

OnlyIAS Nothing Else Visit :dpp.onlyias.in


Contact : +91-7007 931 912

Q.1) Which of the following statements is/are correct Q.4) Identify the correct statements about Mission for
about the Essential Commodities Act ? Integrated Development of Horticulture (MIDH):
1) There is no specific definition of essential 1) It is a Central Sector Scheme for the holistic growth
commodities in the Essential Commodities Act, of the horticulture sector.
1955. 2) In 2014, CHAMAN ‘Coordinated programme on
2) The Act aims to prevent hoarding and black Horticulture Assessment & Management using
marketing of foodstuffs. geoiNformatics ’ was initiated under MIDH.
3) This act was aimed at curbing inflation 3) It also provides technical advice and administrative
Select the correct answer from the codes given below: support to State Governments/ State Horticulture
a) 1 and 2 only Missions (SHMs) for the Saffron Mission.
b) 1 and 3 only Select the correct answer using the codes given
c) 2 and 3 only below:
d) All of the above a) 1 and 2 only
b) 1 and 3 only
Q.2) Which of the following is/are correct about the c) 2 and 3 only
Agricultural produce market committee (APMC): d) All of the above
1) APMCs are regulated by states.
2) Aim of APMC is to ensure farmers are safeguarded Q.5) Which of the following statements are correct
from exploitation by large retailers, as well as about Operation Green ?
ensuring the farm to retail price spread does not 1) It was launched by the Ministry of Agriculture.
reach excessively high levels 2) Operation Greens seeks to stabilize the supply of
Select the correct answer using the codes given below: Tomato, Onion and Potato (TOP) crops and to
a) 1 only ensure availability of TOP crops throughout the
b) 2 only country round the year without price volatility.
c) Both 1 and 2
3) NAFED will be the Nodal Agency to implement
d) Neither 1 nor 2
price stabilization measures under 'Operations
Green’
Q.3) Which of the following statements about
Select the correct answer from the codes given below:
National Agricultural Cooperative Marketing
a) 1 and 2 only
Federation of India (NAFED) are incorrect?
b) 1 and 3 only
1) One of the objectives of NAFED is to organize,
c) 2 and 3 only
promote, and develop marketing and storage of
d) All of the above
agricultural and forest produce.
2) NAFED is registered under the Multi State Co-
Q.6) Which of the following are the features of the
operative Societies Act.
Model Agriculture Produce and Livestock Marketing
3) NAFED is also one of the procurement agencies
Act, 2017?
along with FCI.
1) Agricultural Produce and Livestock Market
4) NAFED is the Nodal Agency to implement price
Committee (APLMC) provides recognition of a
stabilization measures under Operation Green
State/UT as a single market.
Select the correct answer from the codes given below:
2) To give freedom to the agriculturalists to sell their
a) 2 and 4 only
produce to the buyers and at the place and time of
b) 4 only
their choice.
c) 1 and 3 only
3) Centralisation of Market Committee and State/UT
d) None of the above
Marketing Board to streamline their functions
Select the correct answer from the codes given below:

DPP 2023 DAY 181 1


Contact us :info@onlyias.com

OnlyIAS Nothing Else Visit :dpp.onlyias.in


Contact : +91-7007 931 912

a) 1 and 2 only b) 2 only


b) 1 and 3 only c) Both 1 and 2
c) 2 and 3 only d) Neither 1 nor 2
d) All of the above
Q.10) What does the term “ Seed fund “ mean in the
Q.7) Consider the following statements regarding Indian economy ?
Minimum Support Price: a) Investor invests capital in a startup company in
1. Commission for Agricultural Costs & Prices (CACP) exchange for an equity stake or convertible note
, a statutory body under the Ministry of Agriculture stake in the company
and Farmers Welfare, recommends MSPs for b) Funds provided to industries at times of incurring
mandated crops and fair and remunerative price losses
(FRP) for sugarcane. c) Funds provided for replacement and renovation of
2. The recommendations of the Commission for industries
Agricultural Costs & Prices (CACP) are binding on d) A short-term capital provided to directly aid
the Government. startups with bio technology-based innovations
Which of the statements given above is/ are correct?
a) 1 only Q.11) Consider the following statements regarding
b) 2 only Micro Small and Medium Enterprises:
c) Both 1 and 2 1. The existing criterion of definition of MSMEs is
d) Neither 1 nor 2 based on the Micro Small and Medium Enterprises
development act, 2006 and it is different for
Q.8) India is the largest producer of which of the manufacturing and services units.
following crops? 2. A micro enterprise is an enterprise where the
1. Coffee investment in plant and machinery or equipment
2. Pulses does not exceed ₹1 crore and turnover does not
3. Cotton exceed ₹5 crore
4. Oil palm 3. The National Board for Micro, Small and Medium
Select the correct answer using the code given below: Enterprises (NBMSME) was established by the
a) 1, 2 and 3 only Government without any statutory backing
b) 2 and 3 only Which of the statements given above is/ are not
c) 3 and 4 only correct?
d) 1, 2 ,3 and 4 a) 1 and 2 only
b) 2 and 3 only
Q.9) Consider the following regarding Fisheries sector c) 1 and 3 only
in India: d) 1, 2 and 3
1. While Inland Fisheries are fully managed by State
Governments, Marine Fisheries are a responsibility Q.12) Consider the following statements regarding
of the Central Governments. Index of Industrial Production:
2. The Government of India has extended the facility 1. IIP is an indicator that measures the changes in the
of Kisan Credit Card (KCC) to fisheries and animal volume of production of industrial products during
husbandry farmers to help them meet their a given period, compiled and published monthly by
working capital needs. the National Statistical Office (NSO)
Which of the statements given above is /are not 2. The IIP index is the only measure on the physical
correct? capacity of industrial production.
a) 1 only

DPP 2023 DAY 181 2


Contact us :info@onlyias.com

OnlyIAS Nothing Else Visit :dpp.onlyias.in


Contact : +91-7007 931 912

3. It indicates the performance of eight primary applications which are through approval route is
sectors only , which are the core sectors of the administered by the Ministry of Finance
country's economy Which of the statements given above is/ are correct?
Which of the statements given above is/ are correct? a) 1 only
a) 1 and 2 only b) 2 only
b) 2 and 3 only c) Both 1 and 2
c) 1 and 3 only d) Neither 1 nor 2
d) 1 , 2 and 3
Q.16) Consider the following statements regarding
Q.13) Under the Design Linked Incentive Scheme, Foreign Direct Investments into India :
financial incentives and design infrastructure support 1. Mauritius and Switzerland have emerged as top 2
will be extended to domestic companies, startups and sourcing nations in FDI equity flows into India in
MSMEs. The incentives will be provided across F2021-22 followed by Singapore and USA
various stages of development and deployment of 2. The Top 2 sectors receiving highest FDI Equity
which of the following Industries? Inflow during FY 2021-22 are Computer Software &
a) Textile industry Hardware and Services Sector
b) Semiconductor chips 3. The Top 3 States receiving highest FDI Equity Inflow
c) Defence manufacturing during FY 2021-22 are Maharashtra, Andhra
d) Advanced chemical cells Pradesh and Punjab
Which of the statements given above is/ are not
Q.14) Consider the following statements regarding E- correct?
commerce sector in India : a) 2 only
1. Inventory model of e-commerce means an b) 2 and 3 only
providing of an information technology platform c) 1 and 3 only
by an e-commerce entity on a digital and electronic d) 1, 2 and 3
network to act as a facilitator between buyer and
seller. Q.17) Which one of the following best describes the
2. Marketplace model of e-commerce means an term Asset Monetisation?
ecommerce activity where stocks of goods and a) The public sector companies selling off parts of the
services is owned by e-commerce entity and is sold equity of PSEs to the public
to the consumers directly b) The offering of public infrastructure to the private
Which of the statements given above is/ are correct ? sector or institutional investors through structured
a) 1 only vehicles and mechanisms.
b) 2 only c) The process of converting business from the public
c) Both 1 and 2 sector to the private sector.
d) Neither 1 nor 2 d) The sale of a portion of the Government
Q.15) Consider the following statements regarding shareholding of a public sector enterprise (CPSE) of
Foreign Direct Investment: upto 50%, along with transfer of management
1. Foreign Portfolio Investment is where the foreign control.
entity buys stocks and bonds of a company and
provides the investor with control over the Q.18) Consider the following statements regarding
business. the insurance sector :
2. The Foreign Investment Facilitation Portal (FIFP) 1. Domestic Systemically Important Insurers (D-SIIs)
which facilitates the single window clearance of are identified on an annual basis by Reserve Bank
of India.

DPP 2023 DAY 181 3


Download From - https://upscmaterial.online/

Contact us :info@onlyias.com

OnlyIAS Nothing Else Visit :dpp.onlyias.in


Contact : +91-7007 931 912

2. D-SIIs whose distress or failure would cause a 2. e-NAM provides single window services for all
significant dislocation in the domestic financial Agricultural Produce Market Committee (APMC)
system , are not subjected to any additional related services and information.
regulatory measures. Which of the statements given above is/are correct?
3. Two of the Parameters for identification of D-SIIs a) 1 only
are Size of operations in terms of total revenue and b) 2 only
Global activities across more than one jurisdiction. c) Both 1 and 2
Which of the statements given above is / are not d) Neither 1 nor 2
correct ?
a) 1 and 2 only Q.22) Which of the following are correct about
b) 3 only Pradhan Mantri Fasal Bima Yojana (PMFBY):
c) 1 and 3 only 1) It replaced the National Agricultural Insurance
d) 1, 2 and 3 Scheme (NAIS) and Modified National Agricultural
Insurance Scheme (MNAIS).
Q.19) Consider the following statements regarding 2) The objective of this scheme is to provide
Fincluvation: insurance coverage and financial support to the
1. It is the first-of-its-kind industry initiative to farmers in the event of failure of any of the notified
collaborate with the Fintech Startup community to co- crops as a result of natural calamities, pests &
create and innovate solutions for financial inclusion. diseases.
2. It is launched by the Reserve bank of India (RBI) 3) PMFBY is administered by the Ministry of finance.
under the ongoing Azadi ka Amrit Mahotasav. Select the correct answer from the codes given below:

e
Which of the statements given above is/are Incorrect? a) 1 and 2 only

n
a) 1 only b) 1 and 3 only

li
b) 2 only c) 2 and 3 only
c) Both 1 and 2 d) All of the above

n
d) Neither 1 nor 2 o
Q.23) About fertilizer subsidy in India, which
l.
Q.20) Consider the following statements regarding statements given below is/are correct ?
a

Purchasing Managers Index (PMI) : 1) Indian fertilizer industry is second largest in the
ri

1. PMI is a private sector survey released at the start of world in terms of production and largest in terms
every month and is calculated separately for the of consumption
te

manufacturing and services sectors and then a 2) Urea is the only fertilizer at present with pricing
composite index is constructed. and distribution being controlled statutorily by the
a

2. A score above 50 denotes contraction while one Government.


m

below 50 signifies expansion. Select the correct answer from the codes given below:
c

Which of the statements given above is/are correct? a) 1 only


a) 1 only b) 2 only
s

b) 2 only c) Both 1 and 2


p

c) Both 1 and 2 d) Neither 1 nor 2


.u

d) Neither 1 nor 2
Q.24) Which of the following statements are correct
w

Q.21) Consider the following statements regarding about the production of pulses in Indian agriculture?
w

eNAM: 1) Pulses can be grown in both Kharif and Rabi


1. Food corporation of India is the lead agency for seasons with kharif pulses contributing over 60% of
w

implementing eNAM. the total production.

DPP 2023 DAY 181 4


https://upscmaterial.online/
Download From - https://upscmaterial.online/

Contact us :info@onlyias.com

OnlyIAS Nothing Else Visit :dpp.onlyias.in


Contact : +91-7007 931 912

2) Gram is the most dominant pulse with nearly 40% Q.27) Consider the following statements regarding
share in total pulses production the Bharat Net Project:
3) India is the largest producer, consumer and 1. It is the world’s largest rural connectivity scheme to
importer of pulses in the world be connected by the Optical Fibre network.
Select the correct answer from the codes given below: 2. It is a flagship mission implemented by Bharat
a) 1 and 2 only Broadband Network Limited (BBNL).
b) 1 and 3 only 3. The entire project is being funded by the Universal
c) 2 and 3 only service Obligation Fund (USOF), for improving telecom
d) All of the above services in rural and remote areas of the country.
Which of the Statements given above is/are
Q.25) Which of the following statements are correct Incorrect?
about Pradhan Mantri Annadata Aay Sanrakshan a) 2 only
Abhiyan (PM-AASHA): b) 3 only
1) Price Support Scheme, Price Deficiency Payment c) 1 and 3 only
Scheme, Private Procurement & Stockist Scheme d) None of the above
and Minimum Support Price are the 4 components
of PM-AASHA. Q.28) Consider the following statements regarding
2) The Food Corporation of India (FCI) and the the Lithium manufacturing :
National Agricultural Cooperative Marketing 1. India currently imports all its lithium needs and China
Federation of India (NAFED) will help implement is the world's biggest supplier, with production from
the Price Support Scheme. hard rock mines.

e
3) The Scheme is aimed at ensuring remunerative 2. South Africa, Namibia and Botswana (Lithium

n
prices to the farmers. triangle) in South Africa is believed to account for more

li
Select the correct answer from the codes given below: than 50% of the world's proven Lithium reserves.
a) 1 and 2 only 3. The union government has announced a production-

n
b) 1 and 3 only linked incentive (PLI) scheme to make lithium-ion cells
o
c) 2 and 3 only to promote e-mobility in India.
l.
d) All of the above Which of the Statements given above is/are correct?
a

a) 2 only
ri

Q.26) Consider the following statements regarding b) 3 only


Soil Health Card Scheme: c) 1 and 3 only
te

1. The Soil Health Card Scheme is implemented by the d) All of the above
Central Government in those States and Union
a

Territories which have approved it. Q.29) Consider the following statements Logistics
m

2. It contains the status of soil with respect to 12 Ease Across Different States (LEADS) Index,2021:
c

parameters in which the secondary nutrients are 1. It is a composite indicator to assess international
Nitrogen (N), Phosphorus (P) and Potassium (K). trade logistics across states and Union territories.
s

3. Under the Soil Health Card Scheme, a farmer will 2. Tamil Nadu has been adjudged as the best
p

get the soil health card once every 3 years. performing state since inception of LEADS Index by the
.u

Which of the statements given above is/are correct? ministry of Commerce and Industry.
a) 2 only Which of the statements given above is/are Incorrect?
w

b) 3 only a) 1 only
w

c) 1 and 3 only b) 2 only


d) None of the above c) Both 1 and 2
w

d) Neither 1 nor 2

DPP 2023 DAY 181 5


https://upscmaterial.online/
Download From - https://upscmaterial.online/

Contact us :info@onlyias.com

OnlyIAS Nothing Else Visit :dpp.onlyias.in


Contact : +91-7007 931 912

Which of the Statements given above are correct?


Q.30) Consider the following statements with respect a) 1 and 2 only
to Operating Ratio (OR) in Indian Railways: b) 2 and 3 only
1. Operating Ratio (OR) is the ratio of the working c) 1 and 3 only
expenditure (expenses arising from day-to-day d) All of the above
operations of Railways) to the revenue earned
from traffic. Q.33) Consider the following statements regarding
2. A higher ratio indicates a good ability to generate the Special Economic Zones (SEZs) :
surplus that can be used for capital investments. 1. An SEZ is a territory within a country that is typically
3. A lower ratio indicates a good ability to generate duty-free and has different business and commercial
surplus that can be used for capital investments. laws chiefly to encourage investment and create
4. Operating ratio is a key metric to assess the Indian employment.
Railways financial health. 2. The Baba Kalyani led committee was constituted by
Which of the statement given above is/are Incorrect? the Ministry of Commerce and Industry to study the
a) 2 only existing SEZ policy of India.
b) 3 only 3. Units in SEZs may be allowed to raise external
c) 1 and 3 only commercial borrowing by SEZ units upto US $ 500
d) 1 and 4 only million in a year without any maturity restriction
through recognized banking channels.
Q.31) Consider the following statements regarding Which of the Statements given above are Incorrect?
start-ups ecosystem: a) 2 only

e
1. India is among the top 20 in the global innovation b) 3 only

n
index, 2022 ranking for the first time. c) 2 and 3 only

li
2. India overtook South Korea to emerge as the fourth d) None of the above
highest country in the number of unicorns after the US,

n
UK and China in 2021. Q.34) Consider the following statements regarding
o
3. Start-ups are allowed to accept up to 100% of their the Revamped Distribution Sector Scheme (RDSS) for
l.
funding requirement from foreign venture capital Discoms :
a

investors. 1. It was launched to specifically overcome the


ri

Which of the statements given above is/are correct? operational deficiencies and financial constraints of the
a) 2 only state owned discoms and Private Sector Discoms.
te

b) 3 only 2. It is an umbrella programme merging all the existing


c) 1 and 3 only power sector reforms schemes to increase efficiency.
a

d) All of the above 3. Power Finance Corporation is the sole nodal agency
m

for the implementation of this programme.


c

Q.32) Consider the following statements regarding Which of the Statements given above is/are correct?
the National Rail Plan (NRP) for 2030: a) 1 only
s

1. The NRP is meant to increase the share of railways in b) 2 only


p

freight to 45% by 2030 as a part of a national c) 1 and 3 only


.u

commitment to reduce carbon emission. d) All of the above


2. It assesses locomotive requirements to meet twin
w

objectives of 50% electrification (Green Energy) and Q.35) Which of the following Sectors of the Indian
w

increasing freight modal share. economy where a 100 per cent foreign direct
3. It provides for mapping of the existing railway investment is not permitted under the automatic
w

network on a GIS platform. route ?

DPP 2023 DAY 181 6


https://upscmaterial.online/
Download From - https://upscmaterial.online/

Contact us :info@onlyias.com

OnlyIAS Nothing Else Visit :dpp.onlyias.in


Contact : +91-7007 931 912

1. Pharmaceuticals (Greenfield) 4. IT industry


2. Airports (Brownfield) Select the correct option:
3. Defence manufacturing a) 1 and 2 only
4. Insurance sector b) 3 and 4 only
Select The Correct Answer Using The Codes Given c) 1, 2 and 4 only
Below: d) All of the above
a) 1 and 3 only
b) 2 and 4 only Q.39) Consider the following statements regarding
c) 1, 2 and 3 only the Toy Sector in india :
d) 2, 3 and 4 only 1. The Indian toy sector is mostly fragmented with 90%
of the market being unorganised.
Q.36) Consider the following statements regarding 2. The Indian toys industry has more than 1% of global
the Index of Eight Core Industries (ICI): market share and almost 50% of toys in India are made
1. It is a production volume index that measures the in china.
individual as well as the collective production 3. The Ministry of Education has organized ‘Toycathon’,
performance of eight core industries. a hackathon to develop toys and games based on
2. The eight-core industries have a total weight of Indian culture and ethos.
nearly 50 percent in the Index of Industrial Production. Which of the Statements given above are correct?
3. The compilation and releasing of the index are done a) 1 and 2 only
by the Office of the Economic Adviser (OEA) with the b) 2 and 3 only
base year 2011-12. c) 1 and 3 only

e
Which of the statements given above are correct? d) All of the above

n
a) 1 and 3 only

li
b) 2 and 3 only Q.40) About the Horticulture sector in India, which of
c) 1 and 2 only the following statements is/are correct?

n
d) All of the above 1) Horticulture crops are characterized by high-value
o
crops, higher productivity per unit of area and
l.
Q.37) Consider the following statements regarding lower requirement of irrigation and input cost.
a

Annual Survey of Industries (ASI): 2) India is the largest producer of Horticulture


ri

1. It is the principal source of industrial statistics in products in the world.


India with the objective of estimating the contribution Select the correct answer from the codes given below:
te

of registered manufacturing industries. a) 1 only


2. ASI is computed and published by the Central b) 2 only
a

Statistical Organisation (CSO) on a monthly basis. c) Both 1 and 2


m

Which of the statements given above is/are correct? d) Neither 1 nor 2


c

a) 1 only
b) 2 only Q.41) Consider the following statements about cotton
s

c) Both 1 and 2 production in India and identify the correct


p

d) Neither 1 nor 2 statements:


.u

1) Cotton is a Kharif crop and grows well in black


Q.38) Which of the following industries/sectors are cotton soil of the Deccan plateau.
w

Sunrise Industry ? 2) India is the highest cotton producing country in the


w

1. Space tourism world.


2. Hydrogen fuel production 3) The textile industry employs the highest number of
w

3. Textile industry people in the country after agriculture.

DPP 2023 DAY 181 7


https://upscmaterial.online/
Download From - https://upscmaterial.online/

Contact us :info@onlyias.com

OnlyIAS Nothing Else Visit :dpp.onlyias.in


Contact : +91-7007 931 912

Select the correct answer from the codes given below: Q.45) Which of the following statements regarding
a) 1 and 2 only Paramparagat Krishi Vikas Yojna (PKVY) is/are
b) 1 and 3 only correct?
c) 2 and 3 only 1) It is an extended component of Soil Health
d) All of the above Management (SHM) under the National Mission on
Sustainable Agriculture (NMSA).
Q.42) Which of the following statements regarding 2) With the help of PKVY, the government aims to
the jute industry in India is/are correct? support and promote reduction in dependence on
1) India is the largest producer of jute, also largest in fertilizers and agricultural chemicals
terms of acreage and trade. 3) The funding pattern under the scheme is in the
2) It is mainly concentrated in eastern India because ratio of 50:50 by the Central and State
of the rich alluvial soil of Ganga-Brahmaputra Governments respectively.
delta. Select the correct answer from the codes given below:
Select the correct answer from the codes given below: a) 1 and 2 only
a) 1 only b) 1 and 3 only
b) 2 only c) 2 and 3 only
c) Both 1 and 2 d) All of the above
d) Neither 1 nor 2
Q.46) Which of the following statements are correct
Q.43) About Fair and Remunerative Price (FRP) for about Production Linked Incentive Scheme for Food
sugarcane, which of the following statements are Processing Industry (PLISFPI):

e
correct? 1) One of the objectives of the scheme is to support

n
1) It replaced the Statutory Minimum Price (SMP) of creation of global food manufacturing champions

li
sugarcane in 2009 through amendment of the 2) Apart from inviting foreign companies to set shop
Sugarcane (Control) Order, 1966 in India, the scheme also aims to encourage local

n
2) It is determined on the basis of recommendations companies to set up or expand existing
o
given by the Commission for Agricultural Costs and manufacturing units.
l.
Prices (CACP) 3) It is centrally sponsored scheme launched by
a

3) FRP is generally higher than State-Advised Prices Ministry of Food Processing Industries
ri

(SAPs) Select the correct answer from the codes given below:
Select the correct answer from the codes given below: a) 1 and 2 only
te

a) 1 and 2 only b) 1 and 3 only


b) 1 and 3 only c) 2 and 3 only
a

c) 2 and 3 only d) All of the above


m

d) All of the above


c

Q.47) About Agristack, which of the following


Q.44) Large area certification (LAC) scheme was statements are correct?
s

recently in news is related to 1) AgriStack will create a unified platform for farmers
p

a) Soil health check up to provide them end to end services across the
.u

b) Fertilizer penetration in country agriculture food value chain.


c) Organic farming 2) Under the programme, each farmer will have a
w

d) Identification of total cultivated land unique digital identification


w

3) Access to formal credit, Improved crop insurance


products, Smooth mechanism for marketing and
w

DPP 2023 DAY 181 8


https://upscmaterial.online/
Download From - https://upscmaterial.online/

Contact us :info@onlyias.com

OnlyIAS Nothing Else Visit :dpp.onlyias.in


Contact : +91-7007 931 912

price discovery are some of the benefits of best example in this case. State is prohibited from
Agristack playing this role.
Select the correct answer from the codes given below: 3) Economic roles played by state and private sector
remains fixed
a) 1 and 2 only
Select the correct code among the following
b) 2 and 3 only a) 1 only
c) 1 and 3 only b) 1 and 2 only
d) All of the above c) 2 and 3 only
d) 3 only
Q.48) Consider the following statements regarding
Buffer stocks: Q.51) Consider the following statements regarding
“Income from Abroad” which is taken into account
1. The concept of buffer stock was first introduced
during calculation of GNP
during the 4th Five Year Plan. 1) GNP = GDP + Income from Abroad
2. Buffer stock is used for monthly release of food 2) India’s GDP is always lower than its GNP because
grains for supply through Targeted Public Income from abroad is added which is surplus in
Distribution System (TPDS) and Other Welfare Indian case due to private remittances
Schemes (OWS). 3) Net outcome on the front of the interest payments
on external loan in Indian case has always been
Which of the statements given above is/are correct?
negative as the economy has been a ‘net borrower’
a) 1 only
from the world economies
b) 2 only Which of the above statements are correct?
c) Both 1 and 2 a) 1 and 2 only
d) Neither 1 nor 2 b) 1 and 3 only

e
c) 2 and 3 only
d) All of the above

n
Q.49) Consider the following statements regarding

li
the Rare Earth Elements (REE) :
Q.52) About General Network Access (GNA) which of
1. These are characterized by high density, high melting

n
the following statements is/are correct?
point, high conductivity, and high thermal 1. GNA means open and non-discriminatory access to
o
conductance. the inter-State transmission system
l.
2. REE are crucial inputs for hybrid vehicles, high 2. It ensures each and every contract between power
a

strength magnets and next generation rechargeable producers and the bulk consumers for delivery of
power for smooth supply of electricity
ri

batteries.
Select the correct answer from the codes given below:
3. India has about 37 percent of known global REE
te

a) 1 only
reserves but produces about 60 percent of global REE. b) 2 only
Which of the Statements given above is/are
a

c) Both 1 and 2
Incorrect? d) Neither 1 nor 2
m

a) 1 only
Q.53) Which of the following are components of PM
c

b) 2 and 3 only
Kisan Urja Suraksha evam Utthaan Mahabhiyan
c) 3 only
s

(KUSUM) yojana ?
d) None of the above
p

1. 10,000 MW of decentralised ground-mounted


grid-connected renewable power plants.
.u

Q.50) Which of the following are the characteristics of 2. Installation of standalone solar-powered
Mixed economy agriculture pumps.
w

1) State and private sector both have economic roles. 3. Solarisation of 10 lakh grid-connected solar-
2) Private sector to play those roles where invisible
w

powered agriculture pumps.


hands can work properly. Production and supply of Select the correct code from the codes given below:
the ‘private goods’ (which people use by
w

a) 2 only
purchasing them from their own income) is the b) 1 and 3 only
DPP 2023 DAY 181 9
https://upscmaterial.online/
Download From - https://upscmaterial.online/

Contact us :info@onlyias.com

OnlyIAS Nothing Else Visit :dpp.onlyias.in


Contact : +91-7007 931 912

c) 2 and 3 only 1. PM MITRA park will be developed by a Special


d) All of the above Purpose Vehicle which will be owned by the
Central and State Government in PPP mode
Q.54) Indian Railways have prepared a National Rail 2. Under the scheme, the centre will provide
Plan (NRP) for India – 2030, which of the following are development capital support for the development
its stated objectives? of common infrastructure of Rs 500 crore for each
1. Assess Locomotive requirement to meet twin greenfield MITRA park.
objectives of 100% electrification (Green Energy) Select the correct code from given below
and increasing freight modal share. a) 1 only
2. Reduce transit time of freight substantially by b) 2 only
increasing average speed of freight trains c) Both 1 and 2
3. Singular tracking of congested routes. d) Neither 1 nor 2
Select the correct code from given below:
a) 1 and 2 only Q.58) About FDI statistics in India in 2022, which of
b) 1 and 3 only the following statements is/are incorrect?
c) 2 and 3 only 1. Singapore is the top sourcing nation for FDI in India
d) All of the above 2. The Automobile Industry receives the highest FDi
in India.
Q.55) In one of the following infrastructure financing 3. Gujrat is the top destination to receive FDI
model, 40% of the Project Cost is to be provided by Select the correct code from given below:
the Government as Construction Support during the a) 2 only
construction period and remaining is paid later over b) 1 and 3 only
the period of time and toll fee collection is the c) 2 and 3 only
Government’s responsibility. Which of the following d) None of the above

e
is the model mentioned in this passage
a) Engineering, Procurement, Construction (EPC) Q.59) About Eight Core Sectors in Indian industry

n
model which of the following statements is/are correct ?

li
b) Build-OperateTransfer (BOT) 1. These comprise 40.27% of the weight of items

n
c) Hybrid Annuity Model (HAM) included in the Index of Industrial Production (IIP).
d) Special purpose vehicle (SPV) model 2. Coal has the highest weightage in IIP among all
o
these 8 core sectors.
l.
Q.56) Which of the following statements about the Select the correct code from given below
a

Logistics industry in India is/are correct? a) 1 only


1. A Logistics Division was established under the b) 2 only
ri

Ministry of Road transport and Highways in 2017. c) Both 1 and 2


te

2. PM Gati Shakti was launched as a National Master d) Neither 1 nor 2


Plan for multimodal connectivity to improve
a

logistics Q.60) Which of the following statements are correct


3. Logistics costs have to be cut by half to be near about Agriculture Infrastructure Fund (AIF)
m

global benchmarks by 2030 is the goal of National 1. It is a Central Sector Scheme


Logistics Policy (NLP) 2022 2. The scheme shall provide a medium - long term
c

Select the correct code from given below debt financing facility for investment in viable
s

a) 1 and 2 only projects for post-harvest management


p

b) 1 and 3 only Infrastructure.


c) 2 and 3 only 3. Credit guarantee coverage will be available under
.u

d) All of the above Credit Guarantee Fund Trust for Micro and Small
Enterprises (CGTMSE) scheme
w

Q.57) Which of the following statements is/are Select the correct code from given below
w

correct about PM Mega Integrated Textile Region and a) 1 and 2 only


Apparel (PM MITRA) Parks Scheme? b) 1 and 3 only
w

c) 2 and 3 only
d) All of the above
DPP 2023 DAY 181 10
https://upscmaterial.online/
Download From - https://upscmaterial.online/

Contact us :info@onlyias.com

OnlyIAS Nothing Else Visit :dpp.onlyias.in


Contact : +91-7007 931 912

3. The cost of projects under the Hybrid Annuity


Q.61) Which of the following statements about the Model is inflation-indexed.
National Mission on Edible Oils - Oil Palm (NMEO-OP) Which of the statements given above is/ are correct?
is/are correct?
a) 1 and 2 only
1. It aims to augment the availability of edible oil in
the country by harnessing area expansion, b) 2 and 3 only
increasing crude palm oil production with the aim c) 1 and 3 only
to reduce the import burden. d) 1, 2 and 3
2. It provides Price assurance to the oil palm farmers
on the lines of the MSP Q.64) Consider the following statements regarding
3. It provides special focus on states of western ghats the logistics sector :
due to its favorable climatic factors for the growth
1. NITI Ayog has released the Logistics Ease Across
of palm.
Select the correct code from given below Different States (LEADS) Report aimed at gauging
a) 1 and 2 only the logistics performance of states and identifying
b) 1 and 3 only areas where they can improve logistics
c) 2 and 3 only performance.
d) All of the above 2. India’s logistics costs account for 7-8% of Gross
Domestic Product (GDP), which is on par with
Q.62) Consider the following statements regarding developed countries.
the National Industrial Corridor Programme : Which of the statements given above is/ are correct ?
1. It is a programme in which it is envisaged to a) 1 only
develop new industrial cities as Smart Cities and to b) 2 only

e
converge the next-generation technologies in the c) Both 1 and 2
infrastructure sector.

n
d) Neither 1 nor 2
2. The implementing agency of NICP is the National

li
Highways Authority of India under the Ministry of Q.65) Consider the following statements regarding

n
Road Transport and Highways. the Dedicated Freight Corridor :
3. The main purpose of industrial corridors is the
o
1. Dedicated Freight Corridor (DFC) is a high speed
l.
industrial development and growth, while that of and high capacity railway corridor that is meant for
a

special economic zones (SEZs) are for the the transportation of freight, goods and
promotion of exports.
ri

commodities.
Which of the statements given above is/ are correct ? 2. Western Dedicated Freight Corridor (WDFC) starts
te

a) 2 only at Sahnewal (Ludhiana) in Punjab and ends at


b) 1 and 3 only Dankuni in West Bengal being funded by the World
a

c) 2 and 3 only bank


m

d) 1, 2 and 3 3. The corridors will be constructed by the Indian


c

Railway Catering and Tourism Corporation, a


Q.63) Consider the following statements regarding wholly owned subsidiary of the Indian Railways.
s

the Hybrid Annuity Model : Which of the statements given above is /are not
p

1. Under this Model, the National Highways Authority correct ?


.u

of India (NHAI) pays 40% of the total project a) 1 and 2 only


expenditure and the remaining 60% amount has to b) 2 and 3 only
w

be arranged by the road developer. c) 1 and 3 only


2. The government bears all the Operations and
w

d) 1, 2 and 3
maintenance (O&M) risks and also the full financial
w

risks

DPP 2023 DAY 181 11


https://upscmaterial.online/
Download From - https://upscmaterial.online/

Contact us :info@onlyias.com

OnlyIAS Nothing Else Visit :dpp.onlyias.in


Contact : +91-7007 931 912

Q.66) Consider the following statements regarding 2. The current shareholding pattern of Invest India is
Chip to Start-up (C2S) Programme : 51% of Industry Associations and the remaining
1. Under this programme , the central government 49% of Private sector
seeks applications from start-ups, MSMEs, R&D 3. Invest India shall also be a facilitator and partner
organisations and academia to train engineers in offering hand holding services to the investors to
very-large-scale integration (VLSI) and embedded keep them to speedily fructify their investment
system design areas. plans.
2. The nodal agency for implementation of C2S Which of the statements given above is /are correct ?
programme is National Informatics Centre (NIC) a) 1 and 2 only
under Ministry of Electronics and Information b) 2 and 3 only
Technology. c) 1 and 3 only
Which of the statements given above is/ are correct ? d) 1, 2 and 3
a) 1 only
b) 2 only Q.69) Consider the following statements regarding
c) Both 1 and 2 the Manufacturing and Other Operations in a
d) Neither 1 nor 2 Customs Bonded Warehouse (MOOWR) scheme :
1. MOOWR is a duty exemption scheme in which the
Q.67) Consider the following statements regarding duty on both imported capital goods and inputs
the Remission of Duties and Taxes on Exported stands exempted till their clearance from
Products (RoDTEP) scheme : warehouse.
1. The scheme would refund to exporters the 2. The scheme does not have minimum investment

e
embedded central and state duties but not the requirements or restrictions on its location.

n
local taxes that were so far not being rebated or 3. The scheme is implemented by the Department for

li
refunded but it exempted the local duties or taxes Promotion of Industry and Internal Trade (DPIIT)
from its purview under the Ministry of Commerce and Industry in

n
2. It was started as a replacement for the India. o
Merchandise Export from India Scheme (MEIS), Which of the statements given above is /are correct ?
l.
which was not compliant with the rules of the a) 1 only
a

World Trade Organisation. b) 2 only


ri

3. For Electronics exporters, the Rebate of State and c) 2 and 3 only


Central Levies and Taxes (RoSCTL) Scheme has d) 1, 2 and 3
te

been notified separately.


Which of the statements given above is/ are correct ? Q.70) Consider the following statements regarding
a

a) 2 only the difference between the Consumer Price Index and


m

b) 2 and 3 only the


c

c) 1 and 3 only Wholesale Price Index :


d) 1, 2 and 3 1. The Wholesale Price Index is released by the
s

National Statistical Office (NSO), while Consumer


p

Q.68) Consider the following statements regarding Price Index is Published by the Office of Economic
.u

Invest India : Adviser, Ministry of Commerce and Industry.


1. Invest India was formed in 2009 under Section 25 2. CPI does not capture changes in the prices of
w

of the Companies Act 1956 for promotion of services, which WPI does.
w

foreign investment to build capacity as well as 3. In WPI, more weightage is given to manufactured
bring in global best practices in investment goods, while in CPI, more weightage is given to
w

targeting, promotion and facilitation areas. food items.

DPP 2023 DAY 181 12


https://upscmaterial.online/
Download From - https://upscmaterial.online/

Contact us :info@onlyias.com

OnlyIAS Nothing Else Visit :dpp.onlyias.in


Contact : +91-7007 931 912

Which of the statements given above is /are correct ? promote self-reliance in pharmaceutical
a) 2 only manufacturing.
b) 1 and 2 only Which of the statements given above is/are correct?
c) 3 only a) 1 and 2 only
d) 1, 2 and 3 b) 3 only
c) 2 and 3 only
Q.71) This wildlife sanctuary was found to be one of d) 1 and 3 only
the most suitable areas for the cheetahs as its forests
are not very dense to restrict the fast movement of Q.74) Consider the following statements regarding
the fastest land animal and the prey base for cheetahs Market Based Economic Despatch (MBED)
is also in abundance at the sanctuary. Three-fourths mechanism:
of the wildlife sanctuary falls in the basin of Ganges 1. It proposes a decentralised scheduling of power
tributary, the Yamuna River, and one fourth of the dispatches, both inter-state and intra-state.
sanctuary falls in the Narmada basin.It forms a 2. Power is in the Concurrent List of the Constitution.
corridor between Panna Tiger Reserve and Ratapani 3. It is a way forward to deepen power markets in line
sanctuary. The statement is talking about which with the Centre’s ‘One Nation, One Grid, One
sanctuary? Frequency, One Price’ formula.
a) Gandhi Sagar Wildlife Sanctuary Which of the statements given above is/are correct?
b) Pench Wildlife Sanctuary a) 2 only
c) National Chambal Wildlife Sanctuary. b) 1 and 2 only
d) Nauradehi Wildlife Sanctuary c) 1 and 3 only

e
d) 2 and 3 only

n
Q.72) Consider the following statements regarding

li
Climate Reparations: Q.75) Consider the following statements:
1. It refers to a call for money to be paid by the Global This commercial plant is equatorial in nature and needs

n
North to the Global South as Global North is above 25°C with a moist and humid climate and rainfall
o
responsible for 92% of excess global carbon emissions. more than 200 cm. It grows well-drained alluvial soil.
l.
2. It is an extension of the universally acknowledged The British established its first plantation in India in
a

“Polluter Pays” principle. 1902 on the banks of the river Periyar in Kerala. 100%
ri

Which of the statements given above is/are Incorrect? Foreign Direct Investment (FDI) is allowed in its
a) 1 only plantations.
te

b) 2 only Which of the options correctly explains the


c) Both 1 and 2 information given above?
a

d) Neither 1 nor 2 a) Coconut


m

b) Coffee
c

Q.73) Consider the following statements regarding c) Rubber


Scheme for the Promotion of Bulk Drug Parks: d) Mustard
s

1. It aims to develop 7 mega Bulk Drug parks in India


p

with Grants-in-Aid of maximum Rs. 7000 Crore per Bulk Q.76) What are the benefits of Direct Benefit Transfer
.u

Drug Park. (DBT) in an economy:


2. The scheme will be implemented by the Department 1. Promotes accountability in subsidy distribution
w

of Pharmaceuticals, ministry of chemical fertilizers. 2. Increases pilferage in the distribution of money and
w

3. The facilities provided under this scheme will bring reduces the misuse of public funds
down the manufacturing cost of the bulk drugs and will 3. Increasing transparency through eliminating ghost
w

beneficiaries

DPP 2023 DAY 181 13


https://upscmaterial.online/
Download From - https://upscmaterial.online/

Contact us :info@onlyias.com

OnlyIAS Nothing Else Visit :dpp.onlyias.in


Contact : +91-7007 931 912

4. Save time and cost by increasing intermediaries Q.80) Consider the following statements in context of
Which of the options given above is/are Incorrect? bail provisions under Section 437 of the Code of
a) 3 only Criminal Procedure (CrPC):
b) 2 and 4 only 1. Section 437 of the CrPC deals with bail in case of
c) 1, 2 and 4 only bailable and non-bailable offences.
d) 1 and 3 only 2. It contains an exception in a provision that says the
court may grant bail even in these cases, “if such
Q.77) Consider the following statements regarding person is under the age of 16 or is a woman or is sick
Human Trafficking: or infirm”.
1. There is no law for prevention of trafficking for Which of the statements given above is/are Incorrect?
commercial sexual exploitation in india. a) 1 only
2. India has not ratified the United Nations Convention b) 2 only
on Transnational Organised Crime (UNCTOC) yet. c) Both 1 and 2
Which of the statements given above is/are Incorrect? d) Neither 1 nor 2
a) 1 only
b) 2 only Q.81) Which of the following article of constitution is
c) Both 1 and 2 related to ‘doctrine of essentiality’ recently used by
d) Neither 1 nor 2 the courts?
a) Article 19
Q.78) Recently seen in news “Operation Megha b) Article 21
Chakra” is associated with? c) Article 25

e
a) To track down illegal poaching and trafficking of d) Article 32

n
wild animals

li
b) A military-operation carried out by Indian security Q.82) Consider the following statements regarding
forces to deal with insurgents Netaji Subhash Chandra Bose:

n
c) Crackdown on online circulation and sharing of 1. Recently Netaji Subhash Chandra Bose monolithic
o
child sexual abuse material statue was inaugurated at Kartavya Path.
l.
d) Initiative to bring back citizens stranded in Ukraine 2. Subhash Chandra Bose was the first Indian to join
a

due to Russia-Ukraine war the Indian Civil Service.


ri

3. He led the foundation of the Indian National Army


Q.79) Consider the following statements regarding with Japanese Major Iwaichi Fujiwara that
te

National Legal Services Authority (NALSA) : comprised Indian prisoners of war.


1. Its main objective is to provide free legal aid and Which of the statements given above is/are correct?
a

advice under Article 40 of the constitution and a) 1 and 2 only


m

organising Lok Adalats. b) 2 only


c

2. The Chief Justice of Supreme Court of India (CJI) is c) 2 and 3 only


the executive chairperson of the Authority. d) 1 only
s

Which of the statements given above is/are Correct?


p

a) 1 only Q.83) Consider the following statements regarding


.u

b) 2 only UN High Sea Treaty:


c) Both 1 and 2 1. The UN High Sea Treaty is also known as the ‘Paris
w

d) Neither 1 nor 2 Agreement for the Ocean’ which deals with


w

‘Biodiversity Beyond National Jurisdiction’.


2. The Treaty excludes the United Nations
w

Convention on Laws of the Sea (UNCLOS).

DPP 2023 DAY 181 14


https://upscmaterial.online/
Download From - https://upscmaterial.online/

Contact us :info@onlyias.com

OnlyIAS Nothing Else Visit :dpp.onlyias.in


Contact : +91-7007 931 912

3. The Treaty's negotiations included establishing ranging in size from micrometers to meters, that
marine protected areas, environmental impact orbit around Saturn
assessments, financial support to countries and b) An optical phenomenon that occurs due to sunlight
sharing other scientific knowledge. refracting in millions of hexagonal ice crystals
Which of the statements given above is/are correct? suspended in the atmosphere.
a) 1 only c) The deformation of the light from a source such as
b) 2 and 3 only a star or galaxy into a ring through gravitational
c) 1 and 3 only lensing of the source’s light by an object with an
d) 1, 2 and 3 extremely large mass.
d) A zone of energetic charged particles, most of
Q.84) Consider the following statements regarding which originate from the solar wind, that are
Ramappa Temple: captured by and held around a planet by that
1. Ramappa Temple, also known as the Rudreshwara planet's magnetosphere
temple, is a UNESCO World Heritage Site located in
the state of Tamilnadu. Q.87) Consider the following statements regarding
2. The temple showcases Dravidian temple Dark Sky Reserve:
architecture and is devoid of any nagara influence. 1. Dark Sky Reserve is a place that has policies to
3. The temple was built by the general of Kakatiya ensure that a tract of land or region has minimal
Ruler, Ganapati Deva. artificial light interference.
Which of the statements given above is/ are correct? 2. Department of Space has announced setting up of
a) 1 and 2 only India’s first dark sky reserve at Hanle in Ladakh

e
b) 1 only 3. Hanle in Ladakh with cloudy skies and high

n
c) 2 and 3 only atmospheric water vapour make it one of the best

li
d) 3 only sites in the world for astronomical observations.
Which of the statements given above is/ are correct?

n
Q.85) Consider the following statements regarding a) 1 only o
Escherichia coli (E.coli) : b) 2 and 3 only
l.
1. E.coli are a large and diverse group of viruses that c) 3 only
a

are found in environment, foods, and intestines of d) 1 , 2 and 3


ri

people and animals


2. Most of the strains of E.coli are harmful and can Q.88) Consider the following statements regarding
te

cause diarrhea, urinary tract infections, respiratory Shrink inflation:


illness and pneumonia, and other illnesses. 1. Shrink inflation is when an economy goes down by
a

Which of the statements given above is / are correct size in monetary terms due to prevailing low prices.
m

? 2. An increase in the cost of input materials and intense


c

a) 1 only competition results in Shrinkflation.


b) 2 only Which of the statements given above is/are Incorrect?
s

c) Both 1 and 2 a) 1 only


p

d) Neither 1 nor 2 b) 2 only


.u

c) Both 1 and 2
Q.86) Which one of the following statements best d) Neither 1 nor 2
w

reflects the idea behind the “Einstein Ring” talked


w

about in the news ? Q.89) Which of the following statements is/are


a) The most extensive ring system of any planet in the correct with respect to Masala Bonds ?
w

Solar System, consist of countless small particles,

DPP 2023 DAY 181 15


https://upscmaterial.online/
Download From - https://upscmaterial.online/

Contact us :info@onlyias.com

OnlyIAS Nothing Else Visit :dpp.onlyias.in


Contact : +91-7007 931 912

1. Masala Bonds are foreign currency-denominated b) 3 only


bonds issued inside India by foreign entities to raise c) 2 only
money in foreign currency from local investors. d) 2 and 3 only
2. The first Masala bond was issued in 2014 by
International Finance Corporation (IFC) for the Q.92) Consider the following statements regarding
infrastructure projects in India. Kushiyara River:
3. These bonds can only be issued to a resident of such 1. Both Brahmaputra and Kushiyara River originate
a country which is a member of the Financial Action from Chemayungdung Glacier.
Task Force (FATF). 2. It acts as a distributary river in Bangladesh and
Select the correct answer using the codes given Assam.
below: Which of the statements given above is/ are correct?
a) 1 and 2 only a) 1 only
b) 3 only b) 2 only
c) 2 and 3 only c) Both 1 and 2
d) 1, 2 and 3 d) Neither 1 nor 2

Q.90) Consider the following statements regarding Q.93) Which of the following are the common
CRISPR Technology: countries that share borders with Kazakhstan and
1. CRISPR-Cas9 technology is often described as Kyrgyzstan both?
‘Genetic Scissors’ which involves the introduction of 1. China
any new gene from the outside. 2. Russia

e
2. Its mechanism is often compared to the ‘cut-copy- 3. Uzbekistan

n
paste’, or ‘find-replace’ functionalities in common 4. Tajikistan

li
computer programmes. 5. Turkmenistan
3. India has approved a 5-year project to develop Which of the options given above is/are correct?

n
CRISPR to cure sickle cell anaemia which will be the first a) 1 and 5 only o
disease that is being targeted for CRISPR-based therapy b) 2, 3 and 4 only
l.
in India. c) 3, 4 and 5 only
a

Which of the statements given above is/are correct? d) 1 and 3 only


ri

a) 1 and 2 only
b) 2 and 3 only Q.94) Consider the following statements regarding
te

c) 3 only International Counter Reassurance Initiative (ICRI)


d) All of the above 1. ICRI is led by India under the leadership of National
a

Security Council Secretariat (NSCS).


m

Q.91) Consider the following statements regarding La 2. Under thematic discussion of the initiative India
c

Nina: leads disruption like disrupt ransomware


1. A “triple-dip” La Nina is a multiyear cooling of the infrastructure and actors.
s

surface temperature of the equatorial Indian 3. CERT-In, under the Ministry of Electronics & IT, is
p

Ocean. the national nodal agency for responding to


.u

2. La Nina will substantially enhance normal to high computer security incidents.


rainfall in the Indian Subcontinent. Which of the statements given above is/ are correct?
w

3. La Nina is opposite to the Positive Indian Ocean a) 1 only


w

Dipole. b) 2 only
Which of the statements given above is/ are correct? c) 1 and 3 only
w

a) 1 and 2 only d) 2 and 3 only

DPP 2023 DAY 181 16


https://upscmaterial.online/
Download From - https://upscmaterial.online/

Contact us :info@onlyias.com

OnlyIAS Nothing Else Visit :dpp.onlyias.in


Contact : +91-7007 931 912

Which of the statements given above is/are Incorrect?


Q.95) Which of the following correctly explain P-75 a) 1 only
recently seen in the news: b) 2 only
a) Newly discovered asteroid c) Both 1 and 2
b) Indigenous construction of submarines d) Neither 1 nor 2
c) Indigenous developed Missile Technology
d) A carbon nanotube Q.99) Consider the following statements in context of
Coral Reefs:
Q.96) Consider the following statements regarding 1. The majority of reef-building corals are found within
Atal Bridge: subtropical waters between 50° north and 50° south
1. Atal bridge is built on the banks of the Sabarmati latitudes.
Riverfront which originates from Mandav Hills. 2. Oxybenzone and Octinoxate are chemicals that
2. Sabarmati River has Luni at its North and Narmada increases coral reef defence against bleaching.
at its South which drains into the Arabian sea. Which of the statements given above is/are Incorrect?
3. Sei, Harnav, Wakal are tributaries of River a) 1 only
Sabarmati. b) 2 only
Which of the statements given above is/are correct? c) Both 1 and 2
a) 1 only d) Neither 1 nor 2
b) 2 and 3 only
c) 3 only Q.100) Consider the following statements regarding
d) 1, 2 and 3 National Clean Air Programme (NCAP):

e
1. Under it 122 non-attainment cities are identified

n
Q.97) Which of the following statement is/are which do not meet the National Ambient Air Quality

li
Incorrect with respect to the Crowding-Out Effect? Standards.
a) It refers to a situation of high government 2. National Ambient Air Quality Standards are set by

n
expenditure supported by high borrowing causing the Central Pollution Control Board (CPCB) under
o
a decrease in private expenditure. Environment protection act,1986.
l.
b) It refers to a situation When an increase in 3. Four air pollutants viz. Sulphur Dioxide (SO2),
a

government spending/investment leads to an Nitrogen Dioxide (NO2), PM10 and PM2.5 have been
ri

expansion of economic activity (real GDP) which in identified only for regular monitoring under National
turn incentivises private sector firms to raise their Ambient Air Quality Standards.
te

own levels of capital investment and employment. Which of the statements given above is/are Incorrect?
c) A high magnitude of the crowding-out effect may a) 1 only
a

even lead to lesser income in the economy. b) 2 and 3 only


m

d) Increased interest rate and reduced availability of c) 1 and 3 only


c

funds increases crowding out effect. d) None of the above


s

Q.98) Consider the following statements regarding


p

Green Fins Hub :


.u

1. The United Nations Environmental Programme


(UNEP), along with UK-based charity Reef-World
w

Foundation has launched the Green Fins Hub.


w

2. Green Fins is a conservation management strategy


that offers the only environmental criteria for maritime
w

tourism that are internationally recognised.

DPP 2023 DAY 181 17


https://upscmaterial.online/
Download From - https://upscmaterial.online/

Contact us :info@onlyias.com

OnlyIAS Nothing Else Visit :dpp.onlyias.in


Contact : +91-7007 931 912

Q.1) Ans: D has Yards/Mandis in the market area that


Exp: regulates the notified agricultural produce and
Essential Commodities Act 1955 livestock. The introduction of APMC was to
● The ECA Act 1955 was legislated at a time when limit the occurrence of Distress Sale by the
the country was facing a scarcity of foodstuffs farmers under the pressure and exploitation of
due to persistent low levels of foodgrains creditors and other intermediaries.APMC
production. ensures worthy prices and timely payments to
● Statement 2 is correct: To prevent hoarding the farmers for their produce.APMC is also
and black marketing of foodstuffs, the responsible for the regulation of agricultural
Essential Commodities Act was enacted in trading practices. This results in multiple
1955. benefits like: Needless intermediaries are
● Statement 1 is correct: There is no specific eliminated, Improved market efficiency
definition of essential commodities in the through a decrease in market charges,The
Essential Commodities Act, 1955. producer-seller interest is well protected.
● Section 2(A) of the Act states that an “essential
commodity” means a commodity specified in
the Schedule of the Act. Q.3) Ans: D
● Statement 3 is correct: The ECA 1955 is used Exp:
to curb inflation by allowing the Centre to All of the statements given are correct, none of them is
enable control by state governments of trade incorrect
in a wide variety of commodities. ● National Agricultural Cooperative Marketing

e
Federation of India (NAFED) was established

n
Extra Edge by OnlyIAS on Gandhi Jayanti on 2 October 1958.

li
● The Act gives powers to the central ● NAFED is registered under the Multi State Co-
government to add or remove a operative Societies Act. It was setup with the

n
commodity in the Schedule. objective to promote cooperative marketing of
o
● The Centre, if it is satisfied that it is agricultural produce to benefit the farmers.
l.
necessary to do so in the public Agricultural farmers are the main members of
a

interest, can notify an item as NAFED, who have the authority in the form of
ri

essential, in consultation with state members of the General Body in the working
governments. of the body.
te

The objectives of NAFED are as follows:


a

Q.2) Ans: C ● To organize, promote, and develop marketing


m

Exp: and storage of agricultural and forest


● Both Statements are correct: An Agricultural
c

produce.
Produce Market Committee (APMC) is a ● Assist for technical advice in agricultural
s

marketing board established by state production.


p

governments in India to ensure farmers are ● Facilitation, coordination and promotion of


.u

safeguarded from exploitation by large marketing and trading activities of partners


retailers, as well as ensuring the farm to retail associates in the agricultural sector.
w

price spread does not reach excessively high ● Undertaking purchase, sale and supply of
w

levels. APMCs are regulated by states through agricultural, marketing and processing
their adoption of an Agriculture Produce requirements such as manure, seeds, fertilizers
w

Marketing Regulation (APMR) Act. The APMC etc.

DPP 2023 DAY 181 18


https://upscmaterial.online/
Download From - https://upscmaterial.online/

Contact us :info@onlyias.com

OnlyIAS Nothing Else Visit :dpp.onlyias.in


Contact : +91-7007 931 912

● Facilitate the construction of warehouses as development through remote sensing, GIS and
per the Warehousing act by constructing its field survey
own godowns and storage facilities. ● Statement 3 is correct: It also provides
● Act as agent of any government or cooperative technical advice and administrative support to
for the purchase, sale and storage of State Governments/ State Horticulture
agricultural, horticultural and animal Missions (SHMs) for the Saffron Mission and
husbandry produce. other horticulture related activities Rashtriya
● Provide insurance coverage to cover any Krishi Vikas Yojana (RKVY)/National Mission on
accidents that may occur. Sustainable Agriculture (NMSA).
● Organize consultancy work for the benefit of ● Under MIDH, a number of sub schemes and
allied institutions under the NAFED strategies to improve Horticulture production.
● To undertake marketing research and
dissemination of market intelligence;
● To subscribe to the share capital and Q.5) Ans: C
undertake business collaboration with Exp:
cooperative institutions, public, joint and ● Statement 1 is incorrect: The Ministry of Food
private sector enterprises, if and when Processing Industries has launched the
considered necessary for fulfilling the scheme.
objectives of NAFED. ● Statement 2 is correct: Operation Greens
seeks to stabilize the supply of Tomato, Onion
The NAFED is part of “Operation Greens” where price and Potato (TOP) crops and to ensure

e
stabilization measures are implemented in order to availability of TOP crops throughout the

n
increase farmers’ income by 2022. country round the year without price volatility.

li
NAFED also take part in procurement of foodgrains The scheme was extended during June 2020 to
along with FCI cover all fruits & vegetables (TOTAL) for a

n
period of six months on pilot basis as part of
o
Aatmanirbhar Bharat Abhiyan.
l.
● Statement 3 is correct: NAFED will be the
a

Q.4) Ans: C Nodal Agency to implement price stabilization


ri

Exp: measures under 'Operations Green’.


Mission for Integrated Development of Horticulture
te

(MIDH) Extra Edge by OnlyIAS


● Statement 1 is incorrect: It is a Centrally
a

Sponsored Scheme for the holistic growth of In the budget speech of 2018-19, a new Scheme
m

the horticulture sector. “Operation Greens” was announced on the line of


c

○ Under MIDH, Government of India “Operation Flood”, with an outlay of Rs. 500 crore to
(GOI) contributes 60% of total outlay promote Farmer Producers Organizations, agri-
s

for developmental programmes in all logistics, processing facilities and professional


p

the states (90% for states in North East management.


.u

and Himalayas).
● Statement 2 is correct: In 2014, CHAMAN Objectives
w

‘Coordinated programme on Horticulture ● Enhancing value realization of TOP farmers by


w

Assessment & Management using targeted interventions to strengthen TOP


geoiNformatics ’ was initiated under MIDH for production clusters and their FPOs, and
w

better horticulture assessment and linking/connecting them with the market.

DPP 2023 DAY 181 19


https://upscmaterial.online/
Download From - https://upscmaterial.online/

Contact us :info@onlyias.com

OnlyIAS Nothing Else Visit :dpp.onlyias.in


Contact : +91-7007 931 912

● Price stabilization for producers and regulatory functions, while the latter
consumers by proper production planning in would have to look after the
the TOP clusters and introduction of dual use developmental responsibilities under
varieties. the Act.
● Reduction in post-harvest losses by creation of ○ Creation of a conducive environment
farm gate infrastructure, development of to set up operative private wholesale
suitable agro-logistics, creation of appropriate market yards and farmer-consumer
storage capacity linking consumption centers. market yards, to enhance competition
● Increase in food processing capacities and among different markets.
value addition in TOP value chain with firm ○ Promoting direct interface between
linkages with production clusters. farmers and processors/bulk-
● Setting up of a market intelligence network to buyers/exporters/end users to reduce
collect and collate real time data on demand the price spread to benefit both
and supply and price of TOP crops. producers and consumers.
○ Enable declaration of
warehouses/silos/cold storages and
Q.6) Ans: A other structure/space as market sub-
Exp: yard to provide better market
The features of the Model APLM Act, 2017 are listed access/linkages to farmers.
below: ● Statement 2 is correct: To give freedom to the
● Statement 1 is correct: For abolition of agriculturalists to sell their produce to the

e
fragmentation of market within the buyers and at the place and time of their

n
State/Union Territory (UT) by removing the choice.

li
concept of ‘notified market area’ in the ○ To promote e-trading to improve
regulation of Agricultural Produce and transparency in trade operations and

n
Livestock Market Committee (APLMC). In integration of markets across
o
simple terms, APLM provides recognition of a geographies.
l.
State/UT as a single market. ○ To provide provisions for single point
a

○ In addition to cereals, pulses and levy of market fee across the State and
ri

oilseeds, the Act provides geographical unified single trading license to realize
restriction-free trade transaction of cost-effective transactions.
te

agricultural produce including ○ To promote a national market for


commercial crops like cotton, agricultural produce through
a

horticulture crops, livestock, fisheries provisioning of an inter-state trading


m

and poultry. license, grading and standardization


c

○ Disintermediation of the food supply and quality certification.


chain by the integration of farmers, ○ To rationalize market fee and
s

exporters, processors, bulk retailers commission charges.


p

and consumers. ○ Grants provision for Special


.u

○ There is a clear separation of functions Commodity market yards and Market


and powers between the Director of yards of National Importance (MNI).
w

Agricultural Marketing and the ● Statement 3 is incorrect: Complete


w

Managing Director of State/UT democratization of Market Committee and


Agricultural Marketing Board. The State/UT Marketing Board.
w

former is responsible for carrying out

DPP 2023 DAY 181 20


https://upscmaterial.online/
Download From - https://upscmaterial.online/

Contact us :info@onlyias.com

OnlyIAS Nothing Else Visit :dpp.onlyias.in


Contact : +91-7007 931 912

Q.8) Ans: B
Q.7) Ans: D Exp:
Exp: ● Option 1 is not correct : India is the third-
● Statement 1 is not correct : Commission for largest producer and exporter of coffee in
Agricultural Costs and Prices (CACP) is a Asia and the sixth-largest producer and fifth-
decentralised agency of the Government of largest exporter of coffee in the world. The
India. It was established in 1965 as the country accounts for 3.14% (2019-20) of the
Agricultural Prices Commission, and was given global coffee production. Coffee production in
its present name in 1985. It is an advisory India is dominated in the hill tracts of South
body, not statutory, attached to the Ministry Indian states, with Karnataka accounting for
of Agriculture & Farmers Welfare, 71%, followed by Kerala with 21% and Tamil
Government of India. The commission was Nadu (5%). Indian coffee is said to be the finest
established to recommend Minimum Support coffee grown in the shade rather than in direct
Prices (MSPs), to motivate cultivators and sunlight anywhere in the world. Almost 80% of
farmers to adopt the latest technology in order Indian coffee is exported. The two well-known
to optimise the use of resources and increase species of coffee grown are the Arabica and
productivity. MSP is a “minimum price” for any Robusta. The first variety was introduced in the
crop that the government considers as Baba Budan Giri hill ranges of Karnataka in the
remunerative for farmers and hence deserving 17th century.Brazil is, the largest coffee
of “support”. The CACP is currently composed producer in the world.
of five people. It consists of: A Chairman ; ● Option 2 is correct : Pulses can be grown in

e
Member Secretary ; One Official Member ; Two both Kharif and Rabi seasons with Rabi pulses

n
Non-Official Members ; The two non-official contributing over 60% of the total production.

li
members are usually representatives of the Gram is the most dominant pulse with nearly
farming community and have an active 40% share in total pulses production, followed

n
association with the farming community. by Tur/Arhar and Urad/Black Matpe. Madhya
o
● Statement 2 is not correct : It is an advisory Pradesh, Maharashtra, Rajasthan, Uttar
l.
body whose recommendations are not Pradesh, and Karnataka are the top pulses
a

binding on the Government. The CACP does producing States. Overall, Pulses account for
ri

not do any field-based cost estimates itself. It around 20% of the area under foodgrains and
makes projections using state-wise, crop- contribute around 7-10% of the country’s total
te

specific production cost estimates provided by foodgrains production, making India Largest
the Directorate of Economics & Statistics in the producer (about 25% of global production),
a

Agriculture Ministry. The latter are, however, Largest consumer (about 27% of world
m

generally available with a three-year lag. CACP consumption) and ; Largest importer (about
c

submits its recommendations to the 14%) of pulses in the world.


Government in the form of Price Policy Reports ● Option 3 is correct : India has the distinction of
s

each year. The reports consist of five having the largest area under cotton
p

categories of commodities namely Kharif cultivation which is about 37% of the world
.u

crops, Rabi crops, Sugarcane, Raw Jute, and area under cotton cultivation between 10.5
Copra million hectares to 12.20 million hectares.
w

India is the largest producer of cotton in the


w

Reference: Indian Economy by Vivek Singh- chapter 9- world accounting for about 22% of the world
Agriculture cotton production. The yield per kgshectare
w

which is presently 501 kgs/ha is still lower

DPP 2023 DAY 181 21


https://upscmaterial.online/
Download From - https://upscmaterial.online/

Contact us :info@onlyias.com

OnlyIAS Nothing Else Visit :dpp.onlyias.in


Contact : +91-7007 931 912

against the world average yield of about of the global biodiversity in terms of fish and
789Kgskgs/ha.In the decade ahead till 2030, shellfish species. It contributes nearly 7.7% to
India (25 per cent), China (22 per cent), USA (15 the global fish production and ranks 4 th in
per cent) and Brazil (10 per cent) in that order global exports of fish products with vital role
will continue to dominate the global cotton from states in its governance as Fisheries is a
production which is expected to reach 28.4 State subject. Central Government
million tonnes (mt), while five Asian countries complement’s the former’s efforts under the
— China, India, Pakistan, Bangladesh and guiding principles of cooperative federalism.
Vietnam — will account for 75 per cent of the While Inland Fisheries are fully managed by
total mill consumption (28.3 mt) during the State Governments, Marine Fisheries are a
period, the latest OECD-FAO report on shared responsibility between the Central
Agriculture Outlook 2021-2030 has forecast. and Coastal State/UT Governments.
● Option 4 is not correct : Oil Palm (or Palm Oil) ● Statement 2 is correct : The Government of
is produced from palm fruit of Elaeis India has extended the facility of Kisan Credit
Guineensis (African oil Palm) or Elaeis Oleifera Card (KCC) to fisheries and animal husbandry
(indigenous to South and Central America). As farmers to help them meet their working
a vegetable oil, it is rich in Vitamin A and E with capital needs. The KCC facility will help
no trans fatty acids; used in food products and fisheries and animal husbandry farmers to
in industrial applications. India is the largest meet their short term credit requirements of
palm oil importer in the world. It will reduce rearing of animals, poultry birds, fish, shrimp,
import dependence as India imports 13 million other aquatic organisms and capture of fish.

e
tonnes of edible oil annually out of nearly 25 Under Kisan Credit Card (KCC) facility, for the

n
million tonnes of demand. The domestic existing KCC holders the credit limit is Rs. 3 lakh

li
production of crude palm oil (CPO) is expected including animal husbandry and fisheries
to go up to 11.20 lakh tonnes by 2025-26 and activities whereas the KCC holders for animal

n
up to 28 lakh tonnes by 2029-30.84% of global husbandry and fisheries have the credit limit of
o
palm oil production comes from Indonesia and Rs. 2 lakh to meet their working capital
l.
Malaysia. Other producers include Thailand, requirements for animal husbandry and
a

Colombia, Nigeria, Guatemala, and Ecuador. fisheries activities. Under KCC facility, Interest
ri

subvention is available for animal husbandry


Reference : https://www.hindustantimes.com/india- and fisheries farmers @ 2% per annum at the
te

news/govt-to-buy-more-pulses-in-push-for-food- time of disbursal of loan and additional interest


security-101661967269009.html subvention @ 3 % per annum in case of prompt
a

repayment as Prompt Repayment Incentive.


m
c

Q.9) Ans: A Reference: Reference: Indian Economy by Vivek Singh-


Exp: chapter 9-
s

● Statement 1 is not correct : India is the 3rd


p

largest fish producing and 2nd largest Q.10) Ans: A


.u

aquaculture nation in the world. Aquaculture Exp:


is the farming of aquatic animals, including ● Option A is correct : Seed money, sometimes
w

finfish, crustaceans, molluscs, etc. and aquatic known as seed funding or seed capital, is a
w

plants, mostly algae, using or within form of securities offering in which an


freshwater, sea water, brackish water, and investor invests capital in a startup company
w

inland saline water. India has more than 10% in exchange for an equity stake or convertible

DPP 2023 DAY 181 22


https://upscmaterial.online/
Download From - https://upscmaterial.online/

Contact us :info@onlyias.com

OnlyIAS Nothing Else Visit :dpp.onlyias.in


Contact : +91-7007 931 912

note stake in the company. The term seed definition was announced in the Atmnirbhar
suggests that this is a very early investment, Bharat package on 13th May, 2020.
meant to support the business until it can ● Statement 2 is correct : As per the
generate cash of its own , or until it is ready for announcement, the definition of Micro
further investments. Seed money options manufacturing and services units was
include friends and family funding, seed increased to Rs. 1 Crore of investment and Rs.
venture capital funds, angel funding, and 5 Crore of turnover. The limit of small unit was
crowdfunding. Traditionally, companies that increased to Rs. 10 Crore of investment and Rs
have yet to meet listing requirements or 50 Crore of turnover. Similarly, the limit of
qualify for bank loans, recognize VC as medium unit was increased to Rs. 20 Crore of
providers of financial support and value added investment and Rs. 100 Crore of turnover. The
services. Seed money can be used to pay for Government of India on 01.06.2020 decided
preliminary operations such as market for further upward revision of the MSME
research and product development. Investors Definition. For medium Enterprises, now it will
can be the founders themselves, using savings be Rs. 50 Crore of investment and Rs. 250
and loans. They can be family members and Crore of turnover.
friends of the founders. Investors can also be ● Statement 3 is not correct : Under the Micro,
outside angel investors, venture capitalists, Small and Medium Enterprises Development
accredited investors, equity crowdfunding Act, 2006, the Government of India
investors, revenue-based financing lenders, or established The National Board for Micro,
government programs. Small and Medium Enterprises (NBMSME) to

e
● Startup India Seed Fund Scheme (SISFS): It examine the factors affecting promotion and

n
aims to provide financial assistance to startups development of MSME. NBMSME has

li
for proof of concept, prototype development, statutory backing ie Micro, Small and
product trials, market entry and Medium Enterprises Development Act, 2006,

n
commercialization. .This board also reviews the existing policies
o
and suggests recommendations to the
l.
Government for the growth of the MSME
a

sector. The National Board for Micro, Small &


ri

Q.11) Ans: C Medium Enterprises (NBMSME) was


Exp: established / notified for the first time on 15th
te

● Statement 1 is not correct : The earlier May 2007 consisting of 47 members including
criterion of definition of Micro Small and Chairman, Vice Chairman and Member
a

Medium Enterprises is based on the MSMED Secretary in accordance with the Sub Section 1
m

Act, 2006. It was different for manufacturing of Section 3 of MSMED Act, 2006 and National
c

and services units. It was also very low in terms Board for Micro, Small & Medium Enterprises
of financial limits. Also, a new composite Rules, 2006. The Minister in-charge of Ministry
s

formula of classification for manufacturing of MSME is ex-officio Chairman of the National


p

and service units has been notified in 2020 . Board.


.u

Now, there will be no difference between


manufacturing and service sectors. Also, a Reference:
w

new criterion of turnover is added.After 14 http://dcmsme.gov.in/BriefNoteNBMSME.htm


w

years since the MSME Development Act came


into existence in 2006, a revision in MSME Q.12) Ans: A
w

Exp:

DPP 2023 DAY 181 23


https://upscmaterial.online/
Download From - https://upscmaterial.online/

Contact us :info@onlyias.com

OnlyIAS Nothing Else Visit :dpp.onlyias.in


Contact : +91-7007 931 912

● Statement 1 is correct : Index of Industrial


Production : IIP is an indicator that measures Q.13) Ans: B
the changes in the volume of production of Exp:
industrial products during a given period. It is ● Option B is correct : The Ministry of
compiled and published monthly by the Electronics and Information (MeitY) is seeking
National Statistical Office (NSO), Ministry of applications from 100 domestic
Statistics and Programme Implementation. It semiconductor chip design firms, companies,
is a composite indicator that measures the start-ups and Micro, Small and Medium
growth rate of industry groups classified Enterprises (MSMEs) under its Design Linked
under: Broad sectors, namely, Mining, Incentive (DLI) Scheme. The DLI scheme is part
Manufacturing, and Electricity. Use-based of the MeitY’s comprehensive Program for the
sectors, namely Basic Goods, Capital Goods, Development of Semiconductors and Display
and Intermediate Goods. Base Year for IIP is Manufacturing Ecosystems in the country.
2011-2012. Significance of IIP: It is used by Lately, there has been an abrupt and cascading
government agencies including the Ministry of shortage of semiconductors worldwide.
Finance, the Reserve Bank of India, etc, for ● Under the DLI Scheme financial incentives and
policy-making purposes.IIP remains extremely design infrastructure support will be
relevant for the calculation of the quarterly extended to domestic companies, startups
and advance GDP (Gross Domestic Product) and MSMEs. The incentives will be provided
estimates. across various stages of development and
● Statement 2 is correct : The Index of Industrial deployment of semiconductor design for

e
Production number gauges the production in Integrated Circuits (ICs), Chipsets, System on

n
various industries for the time under review, Chips (SoCs), Systems & IP Cores and

li
which is generally one month compared to the semiconductor linked design for over a period
reference duration. The IIP index is the only of 5 years. Eligibility: The approved applicants

n
measure of the physical capacity of industrial that claim incentives under the scheme will be
o
production. encouraged to retain their domestic status
l.
● Statement 3 is not correct : Index of Industrial (i.e., more than 50% of the capital in it is
a

Production data or IIP as it is commonly called beneficially owned by resident Indian citizens
ri

is an index that tracks manufacturing activity and/ or Indian companies, which are ultimately
in different sectors of an economy, not only owned and controlled by resident Indian
te

the core sectors . Eight Core Sectors: These citizens) for a period of three years after
comprise only 40.27% of the weight of items claiming incentives under the scheme. An
a

included in the Index of Industrial Production applicant must meet the Threshold and Ceiling
m

(IIP). The eight core sector industries in Limits to be eligible for disbursement of
c

decreasing order of their weightage: Refinery incentives under the Scheme. A dedicated
Products> Electricity> Steel> Coal> Crude Oil> portal has also been made available.
s

Natural Gas> Cement> Fertilizers.It is used by Semiconductors : Any of a class of crystalline


p

government agencies including the Ministry of solids intermediate in electrical conductivity


.u

Finance, the Reserve Bank of India, etc, for between a conductor and an insulator.
policy-making purposes. IIP remains extremely Semiconductors are employed in the
w

relevant for the calculation of the quarterly manufacture of various kinds of electronic
w

and advance GDP estimates. devices, including diodes, transistors, and


integrated circuits. Such devices have found
w

wide application because of their

DPP 2023 DAY 181 24


https://upscmaterial.online/
Download From - https://upscmaterial.online/

Contact us :info@onlyias.com

OnlyIAS Nothing Else Visit :dpp.onlyias.in


Contact : +91-7007 931 912

compactness, reliability, power efficiency, and Inventory, stock management, logistics etc are
low cost. As discrete components, they have not supposed to be actively done by the
found use in power devices, optical sensors, ecommerce firm.
and light emitters, including solid-state lasers.

Reference : Q.15) Ans: D


https://pib.gov.in/PressReleasePage.aspx?PRID=1790 Exp:
346 ● Statement 1 is not correct : Foreign Portfolio
Investors : Foreign portfolio investment (FPI)
consists of securities and other financial assets
Q.14) Ans: D passively held by foreign investors. It does not
Exp: provide the investor with direct ownership of
● Statement 1 is not correct: According to the financial assets and is relatively liquid
FDI policy, “Inventory model of e commerce depending on the volatility of the market.
means an ecommerce activity where where the foreign entity merely buys stocks
inventory of goods and services is owned by and bonds of a company. FPI does not provide
e-commerce entity and is sold to the the investor with control over the
consumers directly.” The main feature of business.Examples of FPIs include stocks,
inventory model is that the customer buys the bonds, mutual funds, exchange traded funds,
product from the ecommerce firm. He American Depositary Receipts (ADRs), and
manages an inventory (stock of products), Global Depositary Receipts (GDRs). FPI is part

e
interfaces with customers, runs logistics and of a country’s capital account and is shown on

n
involves in every aspects of the business. its Balance of Payments (BOP). The BOP

li
Alibaba of China is following the inventory measures the amount of money flowing from
model. one country to other countries over one

n
● Statement 2 is not correct: According to the monetary year. The Securities and Exchange
o
FDI policy guideline, “Marketplace model of e- Board of India (SEBI) brought new FPI
l.
commerce means providing of an information Regulations, 2019, replacing the erstwhile FPI
a

technology platform by an e-commerce entity Regulations of 2014. FPI is often referred to as


ri

on a digital and electronic network to act as a “hot money” because of its tendency to flee at
facilitator between buyer and seller.” the first signs of trouble in an economy. FPI is
te

Marketplaces are platforms that enable a more liquid, volatile and therefore riskier than
large, fragmented base of buyers and sellers to FDI.
a

discover price and transact with one another in ● Statement 2 is not correct : Routes of FDI:
m

an environment that is efficient, transparent Automatic Route : In this, the foreign entity
c

and trusted. The main feature of the market does not require the prior approval of the
place model is that the e-commerce firm like government or the RBI (Reserve Bank of India).
s

flipkart, snapdeal, amazon etc. will be In India FDI up to 100% is allowed in non-
p

providing a platform for customers to interact critical sectors through the automatic route,
.u

with a selected number of sellers. When an not requiring security clearance from the
individual is purchasing a product from flipkart, Ministry of Home Affairs (MHA). Prior
w

he will be actually buying it from a registered government approval or security clearance


w

seller in flipkart. The product is not directly from MHA is required for investments in
sold by flipkart. Here, flipkart is just a website sensitive sectors such as defence, media,
w

platform where a consumer meets a seller. telecommunication, satellites, private security

DPP 2023 DAY 181 25


https://upscmaterial.online/
Download From - https://upscmaterial.online/

Contact us :info@onlyias.com

OnlyIAS Nothing Else Visit :dpp.onlyias.in


Contact : +91-7007 931 912

agencies, civil aviation and mining, besides any either restricted or permitted under the
investment from Pakistan and Bangladesh. Government approval route through a
Government Route: In this, the foreign entity screening mechanism as per the prescribed
has to take the approval of the government. framework.
The Foreign Investment Facilitation Portal ● Statement 1 is not correct : Singapore
(FIFP) facilitates the single window clearance (27.01%) and USA (17.94%) have emerged as
of applications which are through approval top 2 sourcing nations in FDI equity flows into
route. It is administered by the Department India in FY2021-22 followed by Mauritius
for Promotion of Industry and Internal Trade (15.98%), Netherland (7.86%) and Switzerland
(DPIIT), Ministry of Commerce and Industry. (7.31%). It may be noted that as per the
UNCTAD World Investment Report (WIR) 2022,
in its analysis of the global trends in FDI
Q.16) Ans: C inflows, India has improved one position to 7th
Exp: rank among the top 20 host economies for
● India is rapidly emerging as a preferred 2021.
country for foreign investments in the ● Statement 2 is correct : Top 5 sectors receiving
manufacturing sector. FDI Equity inflow in highest FDI Equity Inflow during FY 2021-22
Manufacturing Sectors have increased by 76% are Computer Software & Hardware (24.60%),
in FY 2021-22 (USD 21.34 billion) compared to Services Sector (Fin., Banking, Insurance, Non
previous FY 2020-21 (USD 12.09 billion). The Fin/Business, Outsourcing, R&D, Courier,
Government has implemented several Tech. Testing and Analysis, Other) (12.13%),

e
transformative reforms under the FDI policy Automobile Industry (11.89%), Trading 7.72%

n
regime across sectors such as insurance, and Construction (Infrastructure) Activities

li
defence, telecom, financial services, (5.52%).
pharmaceuticals, retail trading, e-commerce, ● Statement 3 is not correct : Top 5 States

n
construction & development, civil aviation, receiving highest FDI Equity Inflow during FY
o
manufacturing etc. Despite the ongoing 2021-22 are Karnataka (37.55%), Maharashtra
l.
pandemic and global developments, India (26.26%), Delhi (13.93%), Tamil Nadu (5.10%)
a

received the highest annual FDI inflows of and Haryana (4.76%). In India FDI up to 100%
ri

USD 84,835 million in FY 21-22 overtaking last is allowed in non-critical sectors through the
year’s FDI by USD 2.87 billion. Earlier, FDI automatic route, not requiring security
te

inflows increased from USD 74,391 million in clearance from the Ministry of Home Affairs
FY 19-20 to USD 81,973 million in FY 20-21. The (MHA). Prior government approval or security
a

Government continues to liberalize clearance from MHA is required for


m

investment restrictions, eliminate regulatory investments in sensitive sectors such as


c

barriers, nurture international relations and defence, media, telecommunication, satellites,


improve business environment. Changes are private security agencies, civil aviation and
s

made in the FDI policy after having mining, besides any investment from Pakistan
p

consultations with stakeholders including and Bangladesh. All foreign investments are
.u

apex industry chambers, associations, required to be in compliance with the


representatives of industries/groups and applicable entry route, sectoral cap, attendant
w

other organizations. While foreign conditions, sectoral laws, Companies Act, 2013
w

investments are permitted under the and rules thereunder, pricing guidelines,
automatic route in most sectors/activities, due documentation and reporting requirements.
w

to strategic reasons certain investments are

DPP 2023 DAY 181 26


https://upscmaterial.online/
Download From - https://upscmaterial.online/

Contact us :info@onlyias.com

OnlyIAS Nothing Else Visit :dpp.onlyias.in


Contact : +91-7007 931 912

Reference: and increase efficiency, where private


https://pib.gov.in/PressReleasePage.aspx?PRID=1845 companies can move goods quicker and more
719 efficiently
● Option 4 is not correct : Strategic
Q.17) Ans: B disinvestment is the transfer of the ownership
Exp: and control of a public sector entity to some
● Option 1 is not correct : Disinvestment means other entity (mostly to a private sector
sale or liquidation of assets by the entity).Unlike the simple disinvestment,
government, usually Central and state public strategic sale implies a kind of privatization.
sector enterprises, projects, or other fixed The disinvestment commission defines
assets.Privatisation of the public sector strategic sale as the sale of a substantial
companies by selling off parts of the equity of portion of the Government shareholding of a
PSEs to the public is known as disinvestment. central public sector enterprises (CPSE) of upto
The government undertakes disinvestment to 50%, or such higher percentage as the
reduce the fiscal burden on the exchequer, or competent authority may determine, along
to raise money for meeting specific needs, with transfer of management control.
such as to bridge the revenue shortfall from
other regular sources. Reference:
● Option 2 is correct : Asset Monetisation : Also https://m.economictimes.com/news/economy/policy
commonly referred to as asset or capital /asset-monetisation-government-beats-fy22-target-
recycling, it refers to the process of conversion with-rs-96000-crore/articleshow/90807193.cms

e
of assets into economic value. It is a

n
constituent of the government’s non debt

li
capital receipts. Asset monetization, as a
concept, entails offering public infrastructure Q.18) Ans: A

n
to the private sector or institutional investors Exp: o
through structured vehicles and mechanisms. ● Statement 1 is not correct : Domestic
l.
Hence, monetization is distinct shift from Systemically Important Insurers (D-SIIs) refer
a

‘privatization’ or ‘structured partnerships’ with to insurers of such size, market importance and
ri

the private sector within defined contractual domestic and global interconnectedness
frameworks. It serves three critical objectives: whose distress or failure would cause a
te

Unlocks value from public investment in significant dislocation in the domestic financial
infrastructure ; Taps private sector efficiencies system. D-SIIs are identified on an annual
a

; Creation of new sources of revenue by basis by Insurance Regulatory and


m

unlocking of value of hitherto unutilized or Development Authority of India (IRDAI). Their


c

underutilized public assets.Asset monetisation continued functioning is thus critical for the
is the process of creating new sources of uninterrupted availability of insurance services
s

revenue for the government and its entities by to the national economy. They are perceived as
p

unlocking the economic value of unutilised or insurers ‘too big or too important to fail’. IRDAI
.u

underutilised public assets. (Insurance Regulatory and Development


● Option 3 is not correct : Privatization Authority of India) has recently said that LIC,
w

describes the process by which a piece of GIC Re and New India has been retained as
w

property or business goes from being owned Domestic Systemically Important Insurers (D-
by the government to being privately owned. SIIs) for 2021-22.
w

It generally helps governments save money

DPP 2023 DAY 181 27


https://upscmaterial.online/
Download From - https://upscmaterial.online/

Contact us :info@onlyias.com

OnlyIAS Nothing Else Visit :dpp.onlyias.in


Contact : +91-7007 931 912

● Statement 2 is not correct : D-SIIs are ● Statement 1 is correct: Fincluvation is a joint


subjected to additional regulatory measures initiative to collaborate with the Fintech Startup
to deal with the systemic risks and moral community to co-create and innovate solutions
hazard issues. D-SIIs refer to insurers of such for financial inclusion.It is the first-of-its-kind
size, market importance and domestic and industry initiative.It would create a powerful
global interconnectedness, whose distress or platform to mobilize the start-up community
failure would cause a significant dislocation in towards building meaningful financial products
the domestic financial system.D-SIIs are aimed at financial inclusion. The country has
perceived as insurers that are ‘too big or too made rapid strides in the FINTECH space in
important to fail’ (TBTF). This perception and Global Tech world leading innovations like UPI,
the perceived expectation of government Aadhaar. Startups are encouraged to develop
support may amplify risk taking, reduce market solutions aligned with any of the following
discipline, create competitive distortions, and tracks-
increase the possibility of distress in ➢ Creditization: Develop Innovative &
future.These considerations require that D- Inclusive credit products aligned with
SIIs should be subjected to additional the use cases of target customers and
regulatory measures to deal with the systemic take them to their doorsteps through
risks and moral hazard issues. D-SIIs will also Postal network.
be subjected to enhanced regulatory ➢ Digitization: Bring convenience
supervision. through convergence of traditional
● Statement 3 is correct : Parameters for services with Digital Payment

e
identification of D-SIIs are Size of operations Technologies such as making the

n
in terms of total revenue, including premium traditional Money Order service as

li
underwritten and value of assets under Interoperable Banking service.
management ; Global activities across more ➢ Any Market-led solutions that can help

n
than one jurisdiction.The Insurance solve any other problem relevant to
o
Regulatory and Development Authority of IPPB and/or DoP in serving the target
l.
India (IRDAI) has developed a methodology for customers.
a

identification and supervision of D-SIIs. The ● Statement 2 is incorrect: Recently, India Post
ri

parameters include: the size of operations in Payments Bank (IPPB) announced the launch of
terms of total revenue; global activities across
te

Fincluvation under the ongoing Azadi ka Amrit


more than one jurisdiction; lack of Mahotasav. Fincluvation will be a permanent
substitutability of their products and/or
a

platform of IPPB. It will engage startups to


operations; and interconnectedness through
m

participate, ideate, develop and market intuitive


counterparty exposure and macro-economic and tailored products and services for the
c

exposure. customers. It also encourages startups to


s

develop solutions relating to creditization,


Reference:
p

digitization & more. India Post Payments Bank


https://www.thehindu.com/business/Industry/irdai- (IPPB) was launched on September 1, 2018
.u

retains-lic-gic-new-india-as-d-siis/article38074520.ece under the Department of Posts, Ministry of


w

Communication.Its 100% equity is owned by the


Government of India.
w

Q.19) Ans: B
Exp:
w

DPP 2023 DAY 181 28


https://upscmaterial.online/
Download From - https://upscmaterial.online/

Contact us :info@onlyias.com

OnlyIAS Nothing Else Visit :dpp.onlyias.in


Contact : +91-7007 931 912

Reference: help farmers, traders, and buyers with online


https://pib.gov.in/PressReleasePage.aspx?PRID=1818 trading and getting a better price by smooth
635 marketing. Small Farmers Agribusiness
Consortium (SFAC) is the lead agency for
implementing eNAM under the aegis of
Q.20) Ans: A Ministry of Agriculture and Farmers’ Welfare,
Exp: Government of India.
● Statement 1 is correct: PMI is a private sector ● Statement 2 is correct: eNAM portal will
survey-based measure that asks the enable farmers to showcase their products
respondents about changes in their through their nearby markets and facilitate
perception about key business variables as traders from anywhere to quote prices. It
compared with the previous month. It is provides single window services for all
calculated separately for the manufacturing Agricultural Produce Market Committee
and services sectors and then a composite (APMC) related services and information. This
index is constructed. The PMI is usually includes commodity arrivals, quality & prices,
released at the start of every month. It is, buy & sell offers & e-payment settlement
therefore, considered a good leading indicator directly into farmers accounts, among other
of economic activity. Also, since the official services.
data on industrial output, manufacturing and
GDP growth comes much later, PMI helps to Reference: https://www.enam.gov.in/web/
make informed decisions at an earlier

e
stage.PMI provides information about current Q.22) Ans: A

n
and future business conditions to company Exp:

li
decision-makers, analysts, and investors. ● Statement 1 is correct : It replaced the
● Statement 2 is incorrect: The manufacturing National Agricultural Insurance Scheme (NAIS)

n
sector is divided into eight broad categories — and Modified National Agricultural Insurance
o
basic metals, chemicals and plastics, electrical Scheme (MNAIS).
l.
and optical, food and drink, mechanical ● Statement 2 is correct: One of the objectives
a

engineering, textiles and clothing, timber and of PMFBY is to provide insurance coverage and
ri

paper and transport.The PMI is a number from financial support to the farmers in the event of
0 to 100. failure of any of the notified crops as a result of
te

○ PMI above 50 represents an expansion natural calamities, pests & diseases.


when compared to the previous ● Other objectives are:
a

month; ○ To stabilize the income of farmers to


m

○ PMI under 50 represents a ensure their continuance in farming.


c

contraction, and ○ To encourage farmers to adopt


○ A reading at 50 indicates no change. innovative and modern agricultural
s

practices.
p

○ To ensure the flow of credit to the


.u

agriculture sector.
Q.21) Ans: B ● Statement 3 is incorrect: Pradhan Mantri Fasal
w

Exp: Bima Yojana (PMFBY) is administered by the


● Statement 1 is incorrect: E-NAM (National
w

Ministry of Agriculture and Farmers Welfare.


Agriculture Market) is an online trading
w

platform for agriculture produce aiming to Extra Edge by OnlyIAS

DPP 2023 DAY 181 29


https://upscmaterial.online/
Download From - https://upscmaterial.online/

Contact us :info@onlyias.com

OnlyIAS Nothing Else Visit :dpp.onlyias.in


Contact : +91-7007 931 912

Eligibility: Reference: https://www.fert.nic.in/faq#


● Farmers including sharecroppers and tenant
farmers growing notified crops in the notified
areas are eligible for coverage. Q.24) Ans: C
Exp:
Premium: ● Statement 1 is incorrect: Pulses can be grown
● There will be a uniform premium of only 2% to in both Kharif and Rabi seasons with Rabi
be paid by farmers for all Kharif crops and 1.5% pulses contributing over 60% of the total
for all Rabi crops. production.
● In the case of annual commercial and ● Statement 2 is correct: Gram is the most
horticultural crops, the premium to be paid by dominant pulse with nearly 40% share in total
farmers will be only 5%. pulses production, followed by Tur/Arhar and
● The premium rates to be paid by farmers are Urad/Black Matpe. Madhya Pradesh,
very low and the balance premium will be paid Maharashtra, Rajasthan, Uttar Pradesh, and
by the Government to provide a fully insured Karnataka are the top pulses producing States.
amount to the farmers against crop loss on Overall, Pulses account for around 20% of the
account of natural calamities. area under food grains and contribute around
● There is no upper limit on Government 7-10% of the country’s total foodgrains
subsidies. Even if the balance premium is 90%, production.
it will be borne by the Government. ● Statement 3 is correct: India is the largest

e
producer (25% of global production),

n
consumer (27% of world consumption) and

li
Q.23) Ans: B importer (14%) of pulses in the world.
Exp:

n
● Statement 1 is incorrect: Indian fertilizer o
industry is third largest in the world in terms of Q.25) Ans: C
l.
production and second largest in terms of Exp:
a

consumption. Fertilizer production in India has


ri

witnessed a surge over the years, from having ● Statement 1 is incorrect: Price Support
an annual production of 22.23 MMT (million Scheme, Price Deficiency Payment Scheme,
te

metric ton) in 1990-91, it has grown to 43.66 Private Procurement & Stockist Scheme are
MMT in 2021-22. the 3 broad components of PM-AASHA.
a

● Statement 2 is correct: Urea is the only ● Statement 2 is correct: Components of PM-


m

fertilizer at present with pricing and AASHA


1. Price Support Scheme (PSS): Under the PSS,
c

distribution being controlled statutorily by the


Government. The Central Govt. pays subsidies Central nodal agencies will procure pulses,
s

on urea to fertilizer manufacturers on the basis oilseeds and copra with the proactive role of
p

of cost of production at each plant and the state governments.


.u

units are required to sell the fertilizer at the a. The Food corporation of India (FCI) and
government-set Maximum Retail Price (MRP). the National Agricultural Cooperative
w

Thus, no one can sell urea above the MRP Marketing Federation of India (NAFED)
w

declared by the Govt. Under the Concession will help implement the scheme.
Scheme, the MRP for each fertilizer is 2. Price Deficiency Payment Scheme (PDPS):
w

indicative in nature. Under the PDPS, the state will provide the

DPP 2023 DAY 181 30


https://upscmaterial.online/
Download From - https://upscmaterial.online/

Contact us :info@onlyias.com

OnlyIAS Nothing Else Visit :dpp.onlyias.in


Contact : +91-7007 931 912

difference between the prices prevailing in and fertility, low costs and higher profits.
mandis and the MSP. Farmers can track their soil samples and also
3. Private Procurement & Stockist Scheme obtain their Soil Health Card report.
(PPPS): Private agencies will procure oilseeds
in coordination with the government.
● Statement 3 is correct: The Scheme is aimed at Q.27) Ans: D
ensuring remunerative prices to the farmers Exp:
for their produce as announced in the Union • Statement 1 is correct: Bharat Net Project is a
Budget for 2018. The increase in MSP can project envisioned by the Government of India
improve farmer’s income by strengthening to digitally connect all the Gram Panchayats
procurement mechanisms in coordination with (GPs) and Villages of India. It originally aimed
the State Governments. to provide broadband services at 100 Mbps to
around 2.5 lakh gram panchayats of the
country.It is a highly scalable network
Q.26) Ans: B infrastructure accessible on a non-
Exp: discriminatory basis.It is the world’s largest
● Statement 1 is incorrect: Soil Health Card rural connectivity scheme to be connected by
(SHC) Scheme is a Government of India the Optical Fibre network.
initiative promoted by the Department of • Statement 2 is correct: National Optical fibre
Agriculture & Cooperation under the Ministry Network which was launched in October 2011
of Agriculture and Farmers’ Welfare. The was renamed as Bharat Net Project in 2015.It

e
scheme is meant to give each farmer soil is being implemented by the by Bharat

n
nutrient the status of their holding and advise Broadband Network Limited (BBNL). Bharat
Broadband Network Limited is a Special

li
them on the dosage of fertilizers and also the
application of soil amendments that are Purpose Vehicle set up by the Government of

n
needed to maintain soil health in the long run. India under the Companies Act, 1956 with an
o
The scheme is being implemented in all the authorized capital of ₹1000 crore.. It comes
l.
States and Union Territories. under the Ministry of Communications and
a

● Statement 2 is incorrect: Soil Health Card is a Information Technology.


ri

printed report that a farmer will be handed • Statement 3 is correct: The entire project is
over for each of his holdings. It contains the being funded by Universal service Obligation
te

status of soil with respect to 12 parameters Fund (USOF), which was set up for improving
which are as following: telecom services in rural and remote areas of
a

1. Primary Macronutrients: Nitrogen (N), the country.The objective is to facilitate the


m

Phosphorus (P) and Potassium (K). delivery of e-governance, e-health, e-


education, e-banking, Internet and other
c

2. Secondary Nutrients: Sulphur (S)


3. Micronutrients: Zinc (Zn), Boron (B), Iron (Fe), services to rural India.Central Public Sector in
s

Manganese (Mn), Copper (Cu). Undertakings (CPSUs) BSNL, RailTel, and PGCIL
p

4. Physical Parameters: pH, Electrical are providing the optical fibre network for
.u

Conductivity (EC), Organic Carbon (OC). broadband connections for the BharatNet
● Statement 3 is correct: SHC is provided to all project.
w

farmers in the country at an interval of 3 years


w

to enable the farmers to apply recommended Q.28) Ans: B


doses of nutrients based on soil test values to Exp:
w

realize improved and sustainable soil health

DPP 2023 DAY 181 31


https://upscmaterial.online/
Download From - https://upscmaterial.online/

Contact us :info@onlyias.com

OnlyIAS Nothing Else Visit :dpp.onlyias.in


Contact : +91-7007 931 912

• Statement 1 is incorrect: Lithium is the most scheme to make lithium-ion cells to promote e-
sought-after mineral in the past few years on mobility in India. Success of both of them
the back of its usage in battery manufacturing. depends on securing lithium supplies and
Lithium is currently produced from hard rock moving to a greener economy. Demand is
or brine mines. Australia is the world's biggest further expected to rise as Lithium is a key
supplier, with production from hard rock element for new technologies.
mines. Argentina, Chile and China are mainly
producing it from salt lakes.The ancient
igneous rock deposits in Karnataka's Mandya Q.29) Ans: B
district holds the first traces of Lithium ever to Exp:
be discovered in India. The preliminary find is ● Statement 1 is correct: LEADS Index is a
relatively small, a mere 1,600 tonnes of lithium composite indicator to assess international
deposits. India currently imports all its lithium trade logistics across states and Union
needs. Over 165 crore lithium batteries are territories. LEADS is an attempt to bring
estimated to have been imported into India Longevity, Efficiency, Accuracy, Durability, and
between 2016-17 and 2019-20 at an estimated Smartness in the Logistics sector. It is a
import bill of upwards of $3.3 billion. perception-based survey. It is based on a
• Statement 2 is incorrect: Chile, Argentina and stakeholders’ survey for the Ministry of
Bolivia (Lithium triangle) in South America is Commerce and Industry. LEADS is influenced
believed to account for more than 50% of the by the World Bank’s biannual Logistics
world's proven Lithium reserves. Australia and Performance Index (LPI). India’s logistics costs

e
China are the two other geographies that claim account for 13-14 percent of GDP, compared

n
the top spot.In March 2019, India signed a to 7-8 per cent in developed countries.The

li
MoU with Bolivia to explore and extract 2021 index is based on twenty-one indicators
Lithium. India has also signed a bilateral including seventeen perception indicators and

n
agreement with Argentina for securing four objective indicators. o
strategic minerals, which will be ● Statement 2 is incorrect: The Union Minister
l.
operationalized via Khanij Bidesh India Ltd for Commerce and Industry has launched the
a

(KABIL)’s contract with three state-owned 3rd edition of the Logistics Ease Across
ri

organizations in Argentina. India’s first Lithium Different States (LEADS) report, 2021. Gujarat
plant has been set up at Gujarat in 2021, where has been adjudged as the Best Performing
te

a private company has planned investment of State. Haryana second while Punjab got the
third position as per the report. Uttar Pradesh
a

Rs 1000 crore to set up a refinery. The refinery


will use Lithium ore to produce base battery is the top improver. This reflects the efforts
m

material. that have been done in improving the quality


c

• Statement 3 is correct: Lithium is an integral of infrastructure. The first logistic report was
released in 2018. Last year the ranking was not
s

part of the Critical Mineral value chain, whose


importance was clearly highlighted by the released due to the COVID-19 pandemic.
p

COVID causing supply chain disruption. India Gujarat was ranked first in the ranking list both
.u

has unveiled a plan for developing a battery in 2018 and 2019. Gujarat earned the highest
numbers in comparison to any state or union
w

storage ecosystem, which involves setting up


at least 50- gigawatt hour manufacturing territory and has retained its position in the top
w

capacity for advanced chemistry cell batteries. category of Best Performer for the third time in
The union government has also announced a a row
w

₹18,100 crore production-linked incentive (PLI)

DPP 2023 DAY 181 32


https://upscmaterial.online/
Download From - https://upscmaterial.online/

Contact us :info@onlyias.com

OnlyIAS Nothing Else Visit :dpp.onlyias.in


Contact : +91-7007 931 912

Property Organisation (WIPO) showed, India is


Q.30) Ans: A among the top 40, making remarkable
Exp: progress in the global innovation index ranking
● Only Statement 2 is incorrect: The Indian for the first time led by improvement in
Railways has been incurring big losses and information and communication technologies
registered undesirable operating ratios. It (ICT) services exports, venture capital
needs a holistic assessment to improve the recipients’ value, and finance for start-ups.
financial health and operational efficiency. The India is the innovation leader in the lower
Comptroller and Auditor General of India has middle-income group. As per the report, India
tabled the Railways Finances Report in continues to lead the world in the ICT services
Parliament. It has deteriorated from 97.299% exports indicator with the first rank. Recently,
in 2018-19 to 98.36% in 2019-20 Moreover, if India was ranked 40th position out of 132 in
the actual expenditure on pension payments is the Global Innovation Index (GII) 2022 rankings
taken into account, the ratio will be 114.35 %. released by World Intellectual Property
Operating Ratio (OR) is the ratio of the working Organisation (WIPO). India was ranked 46th
expenditure (expenses arising from day-to-day position in 2021.
operations of Railways) to the revenue earned • Statement 2 is incorrect: Start-ups in India
from traffic, a higher ratio indicates a poorer have grown remarkably over the last six years.
ability to generate surplus that can be used The number of new recognised start-ups have
for capital investments such as laying new increased to over 14,000 in 2021-22 from only
lines, or deploying more coaches. The total 733 in 2016-17. As a result, India has become

e
Receipts has decreased by 8.30% in 2019-20 as the third largest start-up ecosystem in the

n
compared to 6.479% increase in 2018- world after the US and China. Further, a record

li
19.Profits from freight traffic were utilised to 44 Indian start-ups have achieved unicorn
compensate for the loss on operation of status in 2021 taking the overall tally of

n
passenger and other coaching services.It unicorns in India to 83, most of these are in the
o
indicates how much the Railways spend to services sector. The UK added only 15 unicorns
l.
earn a rupee. It helps determine the financial to take its total to 39, and hence got displaced
a

health of the Railways.For example, an by India. Unicorn companies are those that
ri

operating ratio of 98.36% for 2019-20, reach a valuation of $1 billion without being
indicates that to earn Rs. 100, the Railways will listed on the stock market and are the dream
te

have to spend Rs. 98.36. Fare-freight ratio is of any tech start-up. India currently has 105
the ratio of the average passenger fare to unicorns and this number is expected to
a

average freight rates. For Indian Railways, it is increase by 140% in the next three years.
m

0.3, compared to 1.4 in South Korea, 1.3 in According to Inc42’s latest ‘The State of Indian
c

France, 1.2 in China, 0.9 in Malaysia and Start-up Ecosystem Report 2022’, India will
Indonesia, and 0.7 in Thailand. have 250 unicorns by 2025, second only to the
s

US, which currently has 618 unicorns.


p
.u

• Statement 3 is correct: Start-ups are allowed


w

to accept up to 100% of their funding


Q.31) Ans: B
requirement from foreign venture capital
Exp:
w

investors. The government is focusing on start-


• Statement 1 is incorrect: Recently, the annual
up companies to promote job creation and
w

report released by the World Intellectual


innovation. The Department for Promotion of

DPP 2023 DAY 181 33


https://upscmaterial.online/
Download From - https://upscmaterial.online/

Contact us :info@onlyias.com

OnlyIAS Nothing Else Visit :dpp.onlyias.in


Contact : +91-7007 931 912

Industry and Internal Trade, which deals with


FDI related matters, compiles all policies
related to foreign investment regime into a Q.33) Ans: D
single document to make it simple and easy for Exp:
investors to understand. • Statement 1 is correct: A Special Economic
Zone or SEZ is a specially marked territory or
enclave within the national borders of a
Q.32) Ans: C country that has more liberal economic laws
Exp: than the rest of the country. An SEZ is an
Statement 1 is correct: Indian Railways have prepared enclave within a country that is typically duty-
a National Rail Plan (NRP) for India – 2030. The Plan is free and has different business and
to create a ‘future ready’ Railway system by 2030. The commercial laws chiefly to encourage
NRP is aimed to formulate strategies based on both investment and create employment. Apart
operational capacities and commercial policy initiatives from generating employment opportunities
to increase modal share of the Railways in freight to and promoting investment, SEZs are created
45% (currently 27%). The objective of the Plan is to also to better administer these areas, thereby
create capacity ahead of demand, which in turn would increasing the ease of doing business. The
also cater to future growth in demand right up to 2050 economic regulations of SEZs tend to be
and also increase the modal share of Railways to 45% conducive to and attract Foreign Direct
in freight traffic and to continue to sustain it. It is Investment (FDI). It facilitates rapid economic
expected that reliance on fossil fuel-based power for growth by leveraging tax incentives to attract

e
the Indian Railways will be significantly reduced by foreign investment and spark technological

n
2030. advancement.

li
Statement 2 is incorrect: As part of the National Rail • Statement 2 is correct: The Special Economic
Plan, Vision 2024 has been launched for accelerated Zones Act was passed in 2005. The Act came

n
implementation of certain critical projects by 2024 into force along with the SEZ Rules in 2006. The
o
such as 100% electrification, multi-tracking of government constituted a committee headed
l.
congested routes, upgradation of speed to 160 kmph by Mr Baba Kalyani, in 2018 to study the
a

on Delhi-Howrah and Delhi-Mumbai routes, existing SEZs of India and prepare a policy
ri

upgradation of speed to 130kmph on all other Golden framework to adopt strategic policy measures.
Quadrilateral-Golden Diagonal (GQ/GD) routes and In the Union Budget this year, the government
te

elimination of all Level Crossings on all GQ/GD proposed to replace the existing law governing
route.NRP assess locomotive requirement to meet
a

Special Economic Zones (SEZs) with a new


twin objectives of 100% electrification (Green Energy) legislation to enable states to become partners
m

by December 2023 and also the increasing traffic right in ‘Development of Enterprise and Service
c

up to 2030 and beyond up to 2050. Hubs’ (DESH).The commerce ministry is


Statement 3 is correct: NRP provides for mapping of proposing a host of direct and indirect
s

the existing railway network on a GIS platform. The incentives such as deferral of import duties and
p

primary value addition of the NRP is an analysis of the exemption from export taxes to revamp
.u

existing network, with expected additions (such as the Special Economic Zones through a new
National Infrastructure Pipeline) also built in. NRP
w

legislation.
bases decision making on objective criteria. It will • Statement 3 is correct: The functioning of the
w

create a future-ready railways system and bring down SEZs is governed by a three-tier administrative
the logistic cost for industry to enable Make in India. setup. The Board of Approval is the apex body
w

DPP 2023 DAY 181 34


https://upscmaterial.online/
Download From - https://upscmaterial.online/

Contact us :info@onlyias.com

OnlyIAS Nothing Else Visit :dpp.onlyias.in


Contact : +91-7007 931 912

and is headed by the Secretary, Department of the supply infrastructure of Discoms. The
Commerce. financial assistance is based on meeting pre-
Major Incentives and Facilities Available to qualifying criteria and upon achievement of
SEZ: basic minimum benchmarks. It is based on the
➢ Duty free import/domestic procurement action plan worked out for each state rather
of goods for development, operation than a ‘one-size-fits-all’ approach. Which will
and maintenance of SEZ units create customized solutions for states.
➢ Exemption from various taxes like • Statement 3 is incorrect: Rural Electrification
Income Tax, minimum alternate tax, etc Corporation and Power Finance Corporation
are the nodal agencies for the implementation
➢ External commercial borrowing by SEZ
of this programme.The scheme will allow
units upto US $ 500 million in a year
reductions in the AT&C losses, improve
without any maturity restriction
financial health,incentivise energy
through recognized banking channels.
conservation & ensure better billing cycle.The
➢ Single window clearance for Central and scheme’s cost is borne by the Centre and state
State level approvals.
governments in a 3:2 ratio. The state
governments will be free to borrow from either
REC-PFC or from other financial institutions to
Q.34) Ans: B
mobilise the funds.It is important to
Exp:
strengthen rural networks to meet the growing
• Statement 1 is incorrect: The Ministry of
demand. Increased supply hours, appliance
Power had launched the Reforms-based and

e
usage and the needs of rural enterprises will
Results-linked, Revamped Distribution Sector

n
need more network investment.
Scheme (RDSS), with the objective to improve

li
the operational efficiencies and financial
Q.35) Ans: D

n
sustainability of State owned DISCOMs/ Power
Exp: o
Departments (excluding Private Sector
● Foreign direct investment or FDI is when an
l.
DISCOMs) by providing financial assistance to
individual or a company invests into the business
DISCOMs for modernisation and strengthening
a

of another country. This investment is


of distribution infrastructure, aiming at
ri

characterized by a notion of direct control.


improvement of the reliability and quality of
foreign direct investment in India is made under
te

supply to end consumers.Revamped


two routes- automatic and approval or
Distribution Sector Reform Scheme (RDSS) has
government.100% Foreign Direct Investment
a

an outlay of Rs 3 lakh crore for five years.Half


(FDI) in pharmaceuticals in India is allowed
m

of the outlay is for better feeder and


under the automatic route for green-field
transformer metering and pre-paid smart
c

pharma.In the case of defence manufacturing,


consumer metering.
the prevailing policy is to allow FDI upto 49%
s

• Statement 2 is correct: RDSS will merge all


through automatic route, but as part of the
p

existing programmes creating an Umbrella


Atmanirbhar Bharat, the government raised this
.u

programme which will increase efficiency.It


limit to 74%. 100% FDI under automatic route in
will merge Integrated Power Development
Greenfield Projects and 74% FDI in Brownfield
w

Scheme,Pradhan Mantri Sahaj Bijli Har Ghar


Projects under automatic route for airports. At
Yojana (PM Saubhagya) and Deen Dayal
w

present, the upper limit for FDI is 74 percent


Upadhyaya Gram Jyoti Yojana.It will provide
into companies that write insurance cover.
w

conditional financial assistance to strengthen

DPP 2023 DAY 181 35


https://upscmaterial.online/
Download From - https://upscmaterial.online/

Contact us :info@onlyias.com

OnlyIAS Nothing Else Visit :dpp.onlyias.in


Contact : +91-7007 931 912

employing 10 or more workers using power


Q.36) Ans: A and those employing 20 or more workers
Exp: without using power.It covers all factories
● Statement 1 is correct: Index of Eight Core registered under the Factories Act, 1948. The
Industries (ICI) refers to a production volume survey also covers Bidi and cigar
index that measures the collective and manufacturing establishments registered
individual production performances of eight under the Bidi and Cigar Workers (Conditions
selected core industries. ICI measures of Employment) Act 1966.Khadi and Village
combined and individual performance of Industries are also included in it.
production in selected eight core industries viz. • Statement 2 is incorrect:The ASI is conducted
Coal, Crude Oil, Natural Gas, Refinery Products, annually under the Collection of Statistics Act,
Fertilizers, Steel, Cement and Electricity. since 1959, to obtain comprehensive and
● Statement 2 is incorrect: Eight Core Sectors detailed statistics of industrial sector with the
comprise 40.27% of the weight of items objective of estimating the contribution of
included in the Index of Industrial Production registered manufacturing industries as a whole
(IIP).The eight core sector industries in to the national income.For the annual revised
decreasing order of their weightage: Refinery estimates, the CSO replaces the IIP with the
Products> Electricity> Steel> Coal> Crude Oil> ASI. The Ministry of Statistics and Programme
Natural Gas> Cement> Fertilizers.Index of Implementation releases the results of the
Industrial Production is an indicator that Annual Survey of Industries (ASI).The Index of
measures the changes in the volume of Industrial Production (IIP) is an index which

e
production of industrial products during a shows the growth rates in different industry

n
given period.It is compiled and published groups of the economy in a stipulated period

li
monthly by the Central Statistical Organization of time. The IIP index is computed and
(CSO), Ministry of Statistics and Programme published by the Central Statistical

n
Implementation. Organisation (CSO) on a monthly basis.
o
● Statement 3 is correct: The compilation and
l.
releasing of the index are done by the Office
a

of the Economic Adviser (OEA), Department


ri

for Promotion of Industry and Internal Trade Q.38) Ans: C


(DPIIT), Ministry of Commerce and Industry. Exp:
te

The new IIP series has revised the base year for ● Sunrise industry is an inherent concept that
the ICI to 2011-12 from the initial base of 2004- shows hope of a rapid boom for a developing
a

05. The calculation of the Index is done by sector or market in its infancy stage. Typically,
m

using the Laspeyres formula. the sunrise industries are characterized by high
c

growth rates, numerous start-ups, and a


wealth of venture capital funding. Such
s

industries create a lot of “buzz” as investors


p

Q.37) Ans: A take an interest in their long-term growth


.u

Exp: prospects and raise public awareness. A


• Statement 1 is correct: Annual Survey of sunrise industry is a new business or business
w

Industries (ASI) is the principal source of sector showing potential for substantial and
w

industrial statistics in India and the most rapid growth.Notable characteristics of sunrise
comprehensive data on organised industries include high-growth rates and a lot
w

manufacturing.It covers all factories of start-ups and venture capital funding.The

DPP 2023 DAY 181 36


https://upscmaterial.online/
Download From - https://upscmaterial.online/

Contact us :info@onlyias.com

OnlyIAS Nothing Else Visit :dpp.onlyias.in


Contact : +91-7007 931 912

cotton textile is not a sunrise industry.The increase exports to the Middle East and African
following are the few examples of the sunrise countries.
industry: • Statement 3 is correct: Favourable
➢ IT industry of California and Bangalore government policies have boosted domestic
➢ Hydrogen fuel production manufacturing and helped the industry to
➢ Petrochemical industry explore global markets and enhance exports.
➢ Food processing industry Increasing customs duties on imported toys
and mandating Bureau of Indian Standards
➢ Space tourism
(BIS) certification requirements for imports
➢ Online Encyclopaedias
have given the necessary push to the Indian toy
industry.Demand for toys based on desi movie
characters, mythological characters and
superheroes such as Chota Bheem are on the
Q.39) Ans: C
rise.Toyotathon 2021 organised by Ministry of
Exp:
Education to crowd- source solutions to
• Statement 1 is correct: The sector is
problems faced by toy industry.National Action
fragmented with 90% of the market being
Plan for Indian Toy Story to boost local
unorganized and 4,000 toy industry units from
manufacturing and incentivise toy and
the MSME sector. 75 percent of domestic
handicraft manufacturers.
manufacturing originates in micro-industries,
while 22 per cent comes from MSMEs. Less
than 3 percent of the domestic toy

e
Q.40) Ans: A
manufacturing processes come from large

n
Exp:
units. Employment in the toy manufacturing

li
● Statement 1 is correct: Horticulture includes
sector offers its female majority workforce
production, utilisation, and improvement of

n
avenues for socio-economic empowerment,
horticultural crops, such as fruits and
o
financial security, and skill development. It also
vegetables, spices and condiments,
l.
offers opportunities for women to act as
ornamental, plantation, medicinal and
agents of change by preserving local toy forms,
a

aromatic plants. Horticulture crops are


intrinsic to their regions.
ri

characterised by high-value crops, higher


• Statement 2 is incorrect: Government’s Make
productivity per unit of area and lower
te

in India has yielded positive results for the toy


requirement of irrigation and input cost.
sector as toy imports in India have declined
● Statement 2 is incorrect: India accounts for
a

70% while exports have increased 61% over


13% of the global production of fruits and 21%
m

the last three years. Indian Toys Industry is


of vegetables, making it the second largest
estimated to be $1.5 bn making up 0.5% of
c

producer, after China. India also ranks at first


global market share. The toy manufacturers in
position in the production of Banana, Mango,
s

India are mostly located in NCR, Maharashtra,


Lime and Lemon, Papaya and Okra. Fruits and
p

Karnataka, Tamil Nadu and clusters across


vegetables account for nearly 90% of total
.u

central Indian states. Almost 85-90% of toys


horticulture production in the country
sold in India are made in China. This is primarily
w

because Chinese toys are sold for a cheap


Q.41) Ans: D
price. The toys industry in India has the
w

Exp:
potential to grow to $2-3 bn by 2024.The
● Statement 1 is correct: Black soil is known as
w

sector is also scouting for new markets to


'black cotton soil' because cotton grows well in

DPP 2023 DAY 181 37


https://upscmaterial.online/
Download From - https://upscmaterial.online/

Contact us :info@onlyias.com

OnlyIAS Nothing Else Visit :dpp.onlyias.in


Contact : +91-7007 931 912

this type of soil. It requires moderate rainfall or Q.43) Ans: A


irrigation and high temperatures. It is mostly Exp:
cultivated in Kharif season and is sown in ● Statement 1 is correct: Fair and Remunerative
August to September. Price (FRP): It replaced the Statutory Minimum
● Statement 2 is correct: India has the Price (SMP) of sugarcane in 2009 through
distinction of having the largest area under amendment of the Sugarcane (Control) Order,
cotton cultivation which is about 37% of the 1966, with FRP as:
world area under cotton cultivation between ○ The minimum price that sugar mills
10.5 million hectares to 12.20 million hectares. have to pay sugarcane farmers to
India is the largest producer of cotton in the insulate them from increasing input
world accounting for about 22% of the world costs.
cotton production. ○ It is linked to a basic recovery rate of
● Statement 3 is correct: The textile industry is sugar, with a premium payable to
second largest after agriculture in India farmers for higher recoveries of sugar
because: (i) Cotton textile industry contributes from sugarcane.
14 percent of the total industrial production. ● Statement 2 is correct: It is determined on the
(ii) It provides employment to 35 million basis of recommendations given by the
persons directly – the second largest after Commission for Agricultural Costs and Prices
agriculture. (CACP), an expert body under the Ministry of
Agriculture & Farmers’ Welfare, and in
Q.42) Ans: B consultation with state governments.

e
Exp: ● Statement 3 is incorrect: State-Advised Prices

n
● Statement 1 is incorrect: In terms of (SAPs) → Used by states to declare SAP for

li
production→ India is the largest producer of sugarcane considering the cost of production
jute followed by Bangladesh and China. and productivity levels. The SAP is generally

n
However, in terms of acreage and trade, higher than the FRP o
Bangladesh takes the lead accounting for
l.
three-fourth of the global jute exports in Q.44) Ans: C
a

comparison to India’s 7%. Exp:


ri

● Statement 2 is correct: It is mainly Large Area Certification’ (LAC) scheme


concentrated in eastern India because of the Under this program the Department of Agriculture,
te

rich alluvial soil of Ganga-Brahmaputra delta. Cooperation and Farmers Welfare (DAC&FW) is
Major jute producing states include West working to identify Traditional Organic Areas to
a

Bengal, Bihar, Odisha, Assam, Andhra Pradesh, transform them into certified organic production
m

Meghalaya and Tripura. hubs.


c

● Under LAC, each village in the area is


Extra Edge by OnlyIAS considered as one cluster/group.
s

Initiatives to promote Jute industry ○ All farmers with their farmland and
p

● Jute-Improved Cultivation and Advanced livestock need to adhere to the


.u

Retting Exercise for improving the productivity standard requirements and on being
and quality of raw jute. verified get certified in mass without
w

● Jute SMART is an e-govt initiative to promote the need to go under conversion


w

transparency in the jute sector. period.


● Jute is included in the MSP regime. ○ Certification is renewed on an annual
w

basis through verification by a process

DPP 2023 DAY 181 38


https://upscmaterial.online/
Download From - https://upscmaterial.online/

Contact us :info@onlyias.com

OnlyIAS Nothing Else Visit :dpp.onlyias.in


Contact : +91-7007 931 912

of peer appraisals as per the process of 2. Strengthen select Indian brand of food
PGS-India. products for global visibility and wider
● The LAC is a quick certification programme acceptance in the international markets;
under the Paramparagat Krishi Vikas Yojana. 3. Increase employment opportunities of off-
farm jobs,
4. Ensuring remunerative prices of farm produce
Q.45) Ans: A and higher income to farmers.
Exp: ● Statement 2 is correct: In order to boost
● Statement 1 and 2 are correct: Paramparagat domestic manufacturing and cut down on
Krishi Vikas Yojana (PKVY) – is a traditional import bills, the central government
farming improvement programme, PMKY was introduced a PLI scheme that aims to give
launched in 2015. It is an extended component companies incentives on incremental sales
of Soil Health Management (SHM) under the from products manufactured in domestic
National Mission on Sustainable Agriculture units. Apart from inviting foreign companies to
(NMSA). With the help of PKVY, the set shop in India, the scheme also aims to
government aims to support and promote: encourage local companies to set up or expand
1. Organic farming existing manufacturing units.
2. Reduction in dependence on fertilizers and ● Statement 3 is incorrect: It is a central sector
agricultural chemicals scheme launched by the Ministry of Food
3. Improvement of the soil health while Processing Industries.
increasing the yields.

e
4. Organic food, thus produced, will be linked

n
with modern marketing tools and local Q.47) Ans: D

li
markets. Exp:
● Statement 1 correct: It is a collection of

n
● Statement 3 is incorrect: Funding pattern → technologies and digital databases that focuses
o
The funding pattern under the scheme is in the on farmers and the agricultural sector.
l.
ratio of 60:40 by the Central and State AgriStack will create a unified platform for
a

Governments respectively. In the case of North farmers to provide them end to end services
ri

Eastern and the Himalayan States, Central across the agriculture food value chain. It is in
Assistance is provided in the ratio of 90:10 line with the Centre’s Digital India programme,
te

(Centre: State). aimed at providing a broader push to digitise


data in India, from land titles to medical
a

records. The government is also implementing


m

Q.46) Ans: A the National Land Records Modernisation


c

Exp: Programme (NRLMP).


● Statement 1 is correct: The objectives of the ● Statement 2 correct: Under the programme,
s

Scheme are to support food manufacturing each farmer will have a unique digital
p

entities with stipulated minimum Sales and identification (farmers’ ID) that contains
.u

willing to make minimum stipulated personal details, information about the land
investment for expansion of processing they farm, as well as production and financial
w

capacity and Branding abroad to incentivise details. Each ID will be linked to the individual's
w

emergence of strong Indian brands: digital national ID Aadhaar.


1. Support creation of global food ● Statement 3 correct: Benefits of Agristack:
w

manufacturing champions; Improved access to formal credit, Improved

DPP 2023 DAY 181 39


https://upscmaterial.online/
Download From - https://upscmaterial.online/

Contact us :info@onlyias.com

OnlyIAS Nothing Else Visit :dpp.onlyias.in


Contact : +91-7007 931 912

crop insurance products, Smooth mechanism able to meet their nutritional


for marketing and price discovery, Better requirements at reasonable prices.
quality of input to address the information
asymmetry,
Q.49) Ans: C
Exp:
Q.48) Ans: C • Statement 1 is correct: The rare-earth
Exp: elements (REE), also called the rare-earth
● Statement 1 is correct: Buffer stock refers to a metals or (in context) rare-earth oxides, or the
reserve of a commodity that is used to offset lanthanides. The rare earth elements (REE) are
price fluctuations and unforeseen a set of seventeen metallic elements. These
emergencies. It is generally maintained for are characterized by high density, high melting
essential commodities and necessities like point, high conductivity, and high thermal
food grains, pulses etc. The concept of buffer conductance. They are considered ‘rare’,
stock was first introduced during the 4th Five because they're scattered in tiny bits all over
Year Plan (1969-74). At present, the the planet, not clustered together in veins, like
Government of India prefers to use the term – other minerals.
Food grain stocking norms – which refers to • Statement 2 is correct: Minerals like Cobalt,
the level of stock in the Central Pool that is Nickel, and Lithium are required for batteries
sufficient to meet the operational requirement used in electric vehicles. REEs are an essential
of food grains and exigencies at any point of — although often tiny — component of more

e
time. Earlier this concept was termed as Buffer than 200 consumer products, including mobile

n
Norms and Strategic Reserve phones, computer hard drives, electric and

li
● Statement 2 is correct: Following are the hybrid vehicles, semiconductors, flatscreen
objectives of Buffer Stock in India: TVs and monitors, and high-end electronics.

n
1. For meeting the prescribed minimum India is seen as a late mover in attempts to
o
buffer stock norms for food security. enter the lithium value chain, coming at a time
l.
2. For monthly release of food grains for when EVs are predicted to be a sector ripe for
a

supply through Targeted Public disruption. The REE are essential raw materials
ri

Distribution System (TPDS) and Other for a wide range of applications, including
Welfare Schemes (OWS). metallurgy (metal refining and metal alloying),
te

3. For meeting emergency situations catalysts in the automotive and the petro-
arising out of unexpected crop failure,
a

chemical industry, colouring of glass/ceramics,


natural disasters, etc. phosphors (LEDs, compact fluorescent lamps,
m

4. For the purpose of Price stabilisation flat panel displays), lasers, rechargeable solid-
c

or market intervention to augment state batteries (Ni-MH), fibre optics and


supply, so as to help moderate the others.
s

open market prices. • Statement 3 is incorrect: China has about 37


p

5. The crops are procured at MSP so that percent of known global REE reserves but
.u

the farmers do not suffer negatively produces about 60 percent of global REE. India
for producing more.
w

has about 6 percent of global REE reserves and


6. In times of deficit, the government produces a miniscule 1 percent of global
w

releases the buffer stocks in a phased supply. India however, has an extremely high
manner so that interests of the possibility of discovery of new deposits along
w

consumers do not suffer, and they are its coastline and hard rock carbonatites that

DPP 2023 DAY 181 40


https://upscmaterial.online/
Download From - https://upscmaterial.online/

Contact us :info@onlyias.com

OnlyIAS Nothing Else Visit :dpp.onlyias.in


Contact : +91-7007 931 912

exist all over the peninsula. China has global Therefore, any output produced by foreign
dominance in REE mining; processing, value residents within the country's borders must be
addition technologies as well as global REE excluded in calculations of GNP, while any
output produced by the country's residents
supply lines. China provides more than 85
outside of its borders must be counted.
percent of the world’s rare earths and it is Whereas gross domestic product GDP
home to about two-thirds of the global supply measures the production of goods and services
of rare metals. made within a country’s borders by both
citizens and foreign residents overall.
● Statement 3 is correct: Interest on External
Q.50) Ans: A Loans→ The net outcome on the front of the
interest payments, i.e., balance of inflow (on
Exp:
the money lend out by the economy) and
● Statement 1 is correct: State and private
outflow (on the money borrowed by the
sector both to have economic roles.
economy) of external interests. In India’s case
● Statement 2 is incorrect: In a mixed economy
it has always been negative as the economy
the private sector plays those roles where the
has been a ‘net borrower’ from the world
invisible hand can work properly. Production
economies.
and supply of the ‘private goods’ (which people
use by purchasing them from their own
income) is the best example in this case. But
state is not prohibited from playing this role. Q.52) Ans: A
● Statement 3 is incorrect: The economic roles Exp:
played by either state or private sector may not ● Statement 1 is correct: GNA means open and
remain fixed for all times to come and may get non-discriminatory access to the inter-State

e
modified as per the needs of the time. transmission system. Technically speaking,

n
Regulation (things like rules, competition, GNA is a non-discriminatory access to the
inter-state transmission system for an

li
taxation, etc.) of the economic system to be
taken care of by the state estimated maximum injection and for a

n
consumer to draw for a specified period.
o
● Statement 2 is incorrect: Keeping with the
concept of “one nation, one grid”, it does away
l.
Q.51) Ans: B
Exp: with unnecessary contracts between power
a

● Statement 1 is correct and statement 2 is producers and the bulk consumers for delivery
ri

incorrect: GNP is commonly calculated by of power. The producer will have the right to
taking the sum of personal consumption access to any consumer as far as the quantum
te

expenditures, private domestic investment, contract is met.


government expenditure, net exports, and any
a

income earned by residents from overseas Extra Edge by OnlyIAS


m

investments, then subtracting income earned


by foreign residents. Central Electricity Authority initiated the idea of GNA
c

1. Gross National Product (GNP) is the GDP of a for:


country added with its ‘income from abroad’. ● Proper planning of transmission system and
s

Here, the trans-boundary economic activities ● Assured recovery of transmission charges from
p

of an economy is also taken into account. the applicant


.u

2. The normal formula is GNP = GDP + Income


from Abroad. But it becomes GNP = GDP + (–
w

Income from Abroad), i.e., GDP – Income from


Abroad, in the case of India. This means that Q.53) Ans: D
w

India’s GNP is always lower than its GDP. Exp:


3. GNP measures the total monetary value of the The PM-KUSUM Scheme was launched in 2019
w

output produced by a country's residents. with 3 components:

DPP 2023 DAY 181 41


https://upscmaterial.online/
Download From - https://upscmaterial.online/

Contact us :info@onlyias.com

OnlyIAS Nothing Else Visit :dpp.onlyias.in


Contact : +91-7007 931 912

● Component-A: For Setting up of 10,000 MW of


Decentralized Grid Connected Renewable ● Formulate strategies based on both
Energy Power Plants on barren land. Under this operational capacities and commercial policy
component, renewable energy based power initiatives to increase modal share of the
plants (REPP) of capacity 500 kW to 2 MW will Railways in freight to 45%.
be setup by individual farmers/ group of ● Statement 2 is correct: Reduce transit time of
farmers/ cooperatives/ panchayats/ Farmer freight substantially by increasing average
Producer Organisations (FPO)/Water User speed of freight trains to 50Kmph.
associations (WUA) on barren/fallow land. ● Statement 3 is incorrect: As part of the
These power plants can also be installed on National Rail Plan, Vision 2024 has been
cultivable land on stilts where crops can also be launched for accelerated implementation of
grown below the solar panels. The renewable certain critical projects by 2024 such as 100%
energy power project will be installed within a electrification, multi-tracking of congested
five km radius of the sub-stations in order to routes, upgradation of speed to 160 kmph on
avoid high cost of sub-transmission lines and to Delhi-Howrah and Delhi-Mumbai routes,
reduce transmission losses. The power upgradation of speed to 130kmph on all other
generated will be purchased by local DISCOM Golden Quadrilateral-Golden Diagonal
at pre-fixed tariff (GQ/GD) routes and elimination of all Level
Crossings on all GQ/GD route.
● Component-B: For Installation of 17.50 Lakh ● Identify new Dedicated Freight Corridors.
stand-alone solar agriculture pumps. Under ● Identify new High Speed Rail Corridors.
this Component, individual farmers will be ● Assess rolling stock requirement for passenger
supported to install standalone solar traffic as well as wagon requirement for

e
Agriculture pumps of capacity up to 7.5 HP for freight.
replacement of existing diesel Agriculture Statement 1 is correct: Assess Locomotive

n

pumps / irrigation systems in off-grid areas, requirement to meet twin objectives of 100%

li
where grid supply is not available. Pumps of electrification (Green Energy) and increasing

n
capacity higher than 7.5 HP can also be freight modal share.
installed, however, the financial support will ● Assess the total investment in capital that
o
be limited to 7.5 HP capacity would be required along with a periodical
l.
break up.
a

● Component-C: For Solarisation of 10 Lakh Grid ● Sustained involvement of the Private Sector in
areas like operations and ownership of rolling
ri

Connected Agriculture Pumps. Under this


Component, individual farmers having grid stock, development of freight and passenger
te

connected agriculture pump will be supported terminals, development/operations of track


to solarise pumps. The farmer will be able to infrastructure etc.
a

use the generated solar power to meet the


irrigation needs and the excess solar power will https://pib.gov.in/PressReleasePage.aspx?PRID=1806
m

be sold to DISCOMs at pre-fixed tariff 617


c

https://www.india.gov.in/spotlight/pm-kusum-
s

pradhan-mantri-kisan-urja-suraksha-evam-utthaan- Q.55) Ans: C


p

mahabhiyan-scheme Exp:
.u

Hybrid Annuity Model (HAM)


Q.54) Ans: A
w

● 40% the Project Cost is to be provided by the


Exp: Government as Construction Support during
w

the construction period and the balance 60%


The key objectives of the National Rail Plan are:- as annuity payments over the operations
w

DPP 2023 DAY 181 42


https://upscmaterial.online/
Download From - https://upscmaterial.online/

Contact us :info@onlyias.com

OnlyIAS Nothing Else Visit :dpp.onlyias.in


Contact : +91-7007 931 912

period along with interest thereon to the logistics cost-to-GDP ratio. The current cost is
concessionaire. 16% of GDP.
● It is a hybrid of EPC and BOT models.
● Toll fee collection is the Government’s
responsibility Q.57) Ans: C
● The objective is to maximize the quantum of Exp:
implemented projects within the available The MITRA park aims to integrate the entire textile
financial resources of the Government and to value chain from spinning, weaving, processing/dyeing,
revive private sector participation in the printing to garment manufacturing at one location.
sector.
● Statement 1 is correct: PM MITRA park will be
https://pib.gov.in/Pressreleaseshare.aspx?PRID=1592 developed by a Special Purpose Vehicle which
673 will be owned by the Central and State
Government and in a Public Private
Partnership (PPP) Mode. Each MITRA Park will
Q.56) Ans: C have an incubation centre, common
Exp: processing house and a common effluent
● Statement 1 is incorrect: A Logistics Division treatment plant and other textile related
was established in the Department of facilities such as design centres and testing
Commerce in 2017. It has been tasked with centres.
undertaking the integrated development of
the Logistics Sector. One of its key ● Statement 2 is correct: Under the scheme, the
responsibility is to develop an action plan to centre will provide development capital
facilitate the overall development of the support for the development of common

e
logistics sector through policy changes, infrastructure of Rs 500 crore for each

n
procedure improvements, identification of greenfield MITRA park and upto Rs 200 crore
bottlenecks and gaps, and adoption of for each brownfield park.

li
technology. The Ministry of Commerce and

n
Industry administers two departments, the
Department of Commerce and the Q.58) Ans: C o
Department for Promotion of Industry & Exp:
l.
Internal Trade. India received the highest annual FDI inflows of USD
a

84,835 million in FY 21-22, overtaking last year’s FDI by


ri

● Statement 2 is correct: In October 2021, PM USD 2.87 billion. In 2021, FDI inflows increased from
Gati Shakti was launched as a National Master USD 74,391 million in FY 19-20 to USD 81,973 million in
te

Plan for multi-modal connectivity to bring FY 20-21. UNCTAD World Investment Report (WIR)
together 16 ministries, including railways and 2022 has ranked India at 7th rank among the top 20
a

roadways, for integrated planning and host economies for 2021, in terms of FDI.
coordinated implementation of infrastructure
m

connectivity projects. Top 5 FDI Sourcing Nation: Statement 1 is correct


c

1. Singapore: 27.01%
● Statement 3 is correct: Recently, the 2. USA: 17.94%
s

Government has launched a National Logistics 3. Mauritius: 15.98%


p

Policy (NLP) 2022, aiming to achieve ‘quick 4. Netherland: 7.86%


.u

last-mile delivery', end transport-related 5. Switzerland: 7.31%


challenges. Logistics costs have to be cut by
w

half to be near global benchmarks by 2030 by Top Sectors: Statement 2 is incorrect


reducing the cost of logistics from 14-18% of 1. Computer Software & Hardware: 24.60%
w

GDP to global best practices of 8%. Countries 2. Services Sector (Fin., Banking, Insurance, Non
like the US, South Korea, Singapore, and Fin/Business, Outsourcing, R&D, Courier, Tech.
w

certain European nations have such a low Testing and Analysis, Other): 12.13%

DPP 2023 DAY 181 43


https://upscmaterial.online/
Download From - https://upscmaterial.online/

Contact us :info@onlyias.com

OnlyIAS Nothing Else Visit :dpp.onlyias.in


Contact : +91-7007 931 912

3. Automobile Industry: 11.89% community farming assets through interest


4. Trading: 7.72% subvention and financial support.
5. Construction (Infrastructure) Activities: 5.52%
● Statement 3 is correct: Benefits under the
Top Destinations: Statement 3 is incorrect scheme
1. Karnataka: 37.55% ● All loans under this financing facility will have
2. Maharashtra: 26.26% interest subvention of 3% per annum up to a
3. Delhi: 13.93% limit of Rs. 2 crore. This subvention will be
4. Tamil Nadu: 5.10% available for a maximum period of seven years.
5. Haryana: 4.76% ● Further, credit guarantee coverage will be
available for eligible borrowers from this
financing facility under Credit Guarantee Fund
Q.59) Ans: A Trust for Micro and Small Enterprises
Exp: (CGTMSE) scheme for a loan up to Rs. 2 crore.
Eight Core Sectors: The fee for this coverage will be paid by the
Coal, Crude Oil, Natural Gas, Refinery Products, Government.
Fertiliser, Steel, Cement, Electricity ● In case of FPOs the credit guarantee may be
● Statement 1 is correct: These comprise 40.27% availed from the facility created under FPO
of the weight of items included in the Index of promotion scheme of Department of
Industrial Production (IIP). Agriculture, Cooperation & Farmers Welfare
● Statement 2 is incorrect: The eight core sector (DACFW).
industries in decreasing order of their ● Moratorium for repayment under this
weightage: financing facility may vary subject to minimum
Refinery Products> Electricity> Steel> Coal> Crude Oil> of 6 months and maximum of 2 years.

e
Natural Gas> Cement> Fertilizers.

n
Extra Edge by OnlyIAS https://vikaspedia.in/schemesall/schemes-for-

li
farmers/agriculture-infrastructure-fund

n
Index of Industrial Production
● The Index of Industrial Production (IIP) is an o
index which details out the growth of various
l.
sectors in an economy such as mineral mining, Q.61) Ans: A
a

electricity, manufacturing, etc. Exp,


ri

● It is compiled and published monthly by the ● Statement 1 is correct: The National Mission
Central Statistical Organisation (CSO), Ministry on Edible Oils - Oil Palm (NMEO-OP) has been
te

of Statistics and Programme Implementation launched with the aim to augment the
six weeks after the reference month ends, i.e., availability of edible oil in the country by
a

a lag of six weeks. harnessing area expansion, increasing crude


m

● The Base Year of the Index of Eight Core palm oil production with the aim to reduce the
Industries has been revised from the year import burden.
c

2004-05 to 2011-12 from April, 2017.


● Statement 2 is correct: It provides Price
s

Q.60) Ans: D assurance to the oil palm farmers on the lines


p

Exp: of the MSP for the Fresh Fruit Bunches (FFBs)


.u

● Statement 1 is correct: Agriculture from which oil is extracted by the industry. This
Infrastructure Fund (AIF) is a Central Sector will be known as the Viability Price (VP).
w

Scheme
● Statement 2 is correct: The scheme shall ● Statement 3 is incorrect: It provides special
w

provide a medium - long term debt financing focus on the Northeast region and the
facility for investment in viable projects for Andaman and Nicobar Islands. To give impetus
w

post-harvest management Infrastructure and to the NorthEast and Andaman, the

DPP 2023 DAY 181 44


https://upscmaterial.online/
Download From - https://upscmaterial.online/

Contact us :info@onlyias.com

OnlyIAS Nothing Else Visit :dpp.onlyias.in


Contact : +91-7007 931 912

Government will additionally bear a cost of 2% ● Statement 1 is correct: The Hybrid Annuity
of the CPO price. Special assistance to North- Model combines two infrastructure
East and the Andaman regions with special construction models, namely the BOT-Annuity
provisions being made for halfmoon terrace
and the EPC. The EPC model comprises 40% of
cultivation, bio fencing and land clearance
along with integrated farming this combination, while the rest, 60%, is BOT-
Annuity.Under the Hybrid Annuity Model, the
National Highways Authority of India (NHAI)
Q.62) Ans: B pays 40% of the total project expenditure.
Exp: This payment is released in ten equal
● Statement 1 is correct : The National instalments based on the completion of
Industrial Corridor Programme is an targeted project milestones. The remaining
ambitious programme of the government of 60% amount has to be arranged by the road
India in which it is envisaged to develop new developer. The developer finances around 20
industrial cities as “Smart Cities” and to to 25% of the total project cost. The remaining
converge the next-generation technologies in amount of money is raised as debt.
the infrastructure sector. Starting with five, ● Statement 2 is not correct : In the new HAM
currently, there are eleven industrial corridors model, a realignment of risk sharing is
that the programme is developing. The idea brought in. The private partner continues to
behind the programme is to create futuristic bear the O&M risks, but the financing risk is
Indian cities that can become global now shared with the government as the
manufacturing and investment destinations. government pays 40% during the construction

e
● Statement 2 is not correct : The implementing stage. The government also shoulders the

n
agency of NICP is the National Industrial responsibility of revenue collection. Hence not
Corridor Development and Implementation

li
one entity takes the burden of all risk and
Trust (NICDIT). NICDIT is under the hence all are comfortable with this model.

n
Department for Promotion of Industry and ● Statement 3 is correct : The Hybrid Annuity
o
Internal Trade (DPIIT). NICDIT originated in Model is a variant of the PPP (Public-Private
l.
2007 as the Delhi Mumbai Industrial Corridor Partnership) model. The cost of projects under
a

Project Implementation Trust Fund (DMIC- the Hybrid Annuity Model is inflation-
ri

PITF). The DMIC-PITF’s mandate was expanded indexed. The change of ownership under the
and redesignated as the NICEDIT in 2016. Hybrid Annuity Model is allowed after six
te

● Statement 3 is correct: The main difference months from the date of commercial
between industrial corridors and special operations. The interest on the Hybrid Annuity
a

economic zones (SEZs) is that while industrial Model projects will be payable on the reducing
m

corridors are for the purpose of industrial balance of project completion cost.
c

development and growth, SEZs are for the


promotion of exports. Currently, there are
s

more than 250 operational SEZs in India, Q.64) Ans: D


p

whereas only 11 industrial corridors are Exp:


.u

envisaged. They are in the developmental ● Statement 1 is not correct : The Ministry of
stage. Commerce and Industry has released the
w

Logistics Ease Across Different States (LEADS)


w

Report (Index). The LEADS report is aimed at


Q.63) Ans: C gauging the logistics performance of states
w

Exp: and Union Territories (UT) and identifying

DPP 2023 DAY 181 45


https://upscmaterial.online/
Download From - https://upscmaterial.online/

Contact us :info@onlyias.com

OnlyIAS Nothing Else Visit :dpp.onlyias.in


Contact : +91-7007 931 912

areas where they can improve logistics The World Bank is funding a majority of the
performance. It was launched in 2018. States EDFC. The 351-km-long ‘New Bhaupur-New
are ranked based on quality and capacity of key Khurja section’ will decongest the existing
infrastructure such as road, rail and Kanpur-Delhi main line and double the speed
warehousing as well as on operational ease of of freight trains from 25 km h to 75 kmph.
logistics including security of cargo, speed of ● Western Dedicated Freight Corridor (WDFC):
terminal services and regulatory approvals. The other arm is the around 1,500-km WDFC
The report is structured along the three from Dadri in Uttar Pradesh to Jawaharlal
dimensions which collectively influence Nehru Port Trust in Mumbai, touching all
logistics ease- Infrastructure, Services, and major ports along the way. The WDFC covers
Operating and Regulatory Environment which Haryana, Rajasthan, Gujarat, Maharashtra and
are further categorised into 17 parameters. Uttar Pradesh. It is being funded by the Japan
● Statement 2 is not correct : India’s logistics International Cooperation Agency.
costs account for 13-14% of Gross Domestic ● Statement 3 is not correct : Constructed by
Product (GDP), compared to 7-8% in Dedicated Freight Corridor Corporation of
developed countries. The government was India Limited (DFCCIL), that has been set up as
aiming to bring down logistics costs by 5% over a special purpose vehicle to build and operate
the next 5 years. Estimated logistics costs in Dedicated Freight Corridors. The Dedicated
India are currently about 14%, which is quite Freight Corridor Corporation of India Limited
high compared to 8-10% globally. Efficient (DFCCIL) is a wholly owned subsidiary of Indian
logistics was pivotal to bring ease and Railways, Ministry of Railways, Government of

e
empowerment to businesses as well as India with the responsibility to undertake

n
citizens. Logistics contributed immensely in planning, development, and mobilisation of

li
our fight against Covid-19 by taking essential financial resources and construction,
supplies including liquid Medical Oxygen maintenance and operation of the "Dedicated

n
throughout the country during the 2nd wave. Freight Corridors" (DFC). Indian Railway
o
Catering and Tourism Corporation (IRCTC) is a
l.
wholly owned subsidiary of the Indian
a

Q.65) Ans: B Railways, Ministry of Railways, Government of


ri

Exp: India. It provides ticketing, catering, and


● Statement 1 is correct : Dedicated Freight tourism services for the Indian Railways.
te

Corridor (DFC): It is a high speed and high


capacity railway corridor that is exclusively
a

meant for the transportation of freight, or in Q.66) Ans: A


m

other words, goods and commodities. DFC Exp:


c

involves the seamless integration of better ● Statement 1 is correct : Under “Chip to Start-
infrastructure and state of the art technology. up (C2S) Programme”, the central
s

● Statement 2 is not correct : Eastern Dedicated government is seeking applications from 100
p

Freight Corridor (EDFC): It starts at Sahnewal start-ups, MSMEs, R&D organisations and
.u

(Ludhiana) in Punjab and ends at Dankuni in academia to train 85,000 engineers in very-
West Bengal. The EDFC route has coal mines, large-scale integration (VLSI) and embedded
w

thermal power plants and industrial cities. system design areas. The C2S programme will
w

Feeder routes are also being made for these. result in development of 175 ASICs
The EDFC route covers Punjab, Haryana, Uttar (application-specific integrated circuits), IP
w

Pradesh, Bihar, Jharkhand and West Bengal core repository and working prototypes of 20

DPP 2023 DAY 181 46


https://upscmaterial.online/
Download From - https://upscmaterial.online/

Contact us :info@onlyias.com

OnlyIAS Nothing Else Visit :dpp.onlyias.in


Contact : +91-7007 931 912

systems on chips (SoC), for a period of five Per capita income is above 1000 USD and
years. It will be a step towards leapfrogging in India’s Per Capita Income crossed above 1000
Electronics System Design & Manufacturing USD in 2017. India subsequently lost the case
(ESDM) space by infusing the culture of at WTO and had to come up with a new WTO
SoC/System Level Design at Bachelors, Masters compliant scheme to help Indian exporters.
and Research level. It will also help in the ● Statement 3 is not correct : For garment
growth of Start-ups involved in fabless design. exporters, the Rebate of State and Central
● Statement 2 is not correct : The nodal agency Levies and Taxes (RoSCTL) Scheme has been
for implementation of C2S programme is C- notified separately.An initiative called the
DAC (Centre for Development of Advanced Rebate of State and Central Levies and Taxes
Computing). C-DAC is a scientific society (RoSCTL) Scheme aims to reimburse all
operating under MeitY. CDAC was created in embedded State and Central Taxes/Levies for
November 1987,[3] initially as the Centre for exports of manufactured goods and garments.
Development of Advanced Computing Importer-Exporter Codes (IECs) are necessary
Technology (C-DACT). The Centre for in order to apply for the RoSCTL programme.
Development of Advanced Computing (C-
DAC) is an Indian autonomous scientific
society, operating under the Ministry of Q.68) Ans: C
Electronics and Information Technology Exp:
● Statement 1 is correct: Invest India was
formed in 2009 under Section 25 of the

e
Q.67) Ans: A Companies Act 1956 for promotion of foreign

n
Exp: investment with 49% equity of the then

li
● Statement 1 is not correct : The Ministry of Department of Industrial Policy and
Commerce and Industry has announced rates Promotion, Ministry of Commerce and

n
of tax refunds under the Remission of Duties Industry and 51% shareholding by FICCI. It
o
and Taxes on Exported Products (RoDTEP) focuses on sector-specific investor targeting
l.
scheme for 8,555 products for the FY 2021-22. and development of new partnerships to
a

The RoDTEP scheme would refund to enable sustainable investments in India. It


ri

exporters the embedded central, state and partners with substantial investment
local duties or taxes that were so far not being promotion agencies and multilateral
te

rebated or refunded and were, therefore, organizations and also actively works with
placing India’s exports at a disadvantage. The several Indian states to build capacity as well
a

rebate under the scheme would not be as bring in global best practices in investment
m

available in respect of duties and taxes already targeting, promotion and facilitation areas.
c

exempted or remitted or credited. ● Statement 2 is not correct : Invest India, a non-


● Statement 2 is correct : It was started in profit company, acts as National Investment
s

January 2021 as a replacement for the Promotion and Facilitation Agency and is the
p

Merchandise Export from India Scheme first point of reference for investors in India.
.u

(MEIS), which was not compliant with the The current shareholding pattern of Invest
rules of the World Trade Organisation. The India is 51% of Industry Associations (i.e.17%
w

MEIS scheme provided additional benefits of each of FICCI, CII & NASSCOM) and the
w

2% to 7% on the Freight On Board (FOB) value remaining 49% of Central and 19 State
of eligible exports. As per the WTO norms, a Governments.
w

country can't give export subsidies like MEIS if

DPP 2023 DAY 181 47


https://upscmaterial.online/
Download From - https://upscmaterial.online/

Contact us :info@onlyias.com

OnlyIAS Nothing Else Visit :dpp.onlyias.in


Contact : +91-7007 931 912

● Statement 3 is correct : Invest India shall act known as the Central Board of Excise and
as a first reference point for investors. Invest Customs (CBEC), is a one of the Boards
India shall also be a facilitator and partner constituted under the statute the Central
offering hand holding services to the investors Boards of Revenue Act, 1963 and is
to keep them to speedily fructify their subordinate to the Department of Revenue
investment plans.Globally, India is one of the under the Ministry of Finance.
most preferred destinations for foreign
investment, however due to the
heterogeneous nature of the Indian market, Q.70) Ans: C
investors do find difficulty in finding clear Exp:
answers to queries and also delays in getting ● Inflation refers to the rise in the prices of most
their investments to fructify speedily. goods and services of daily or common use,
such as food, clothing, housing, recreation,
transport, consumer staples, etc. Inflation
Q.69) Ans: B measures the average price change in a basket
Exp: of commodities and services over time.
● Statement 1 is not correct : Manufacturing Inflation is indicative of the decrease in the
and Other Operations in a Customs Bonded purchasing power of a unit of a country’s
Warehouse is a duty deferment scheme and currency. This could ultimately lead to a
not a duty exemption scheme. The duty on deceleration in economic growth. However, a
both imported capital goods and inputs moderate level of inflation is required in the

e
stands deferred till their clearance from the economy to ensure that production is

n
warehouse. In case of clearance of capital promoted. In India, inflation is primarily

li
goods to DTA, deferred duties will become measured by two main indices — WPI & CPI
payable. In case of clearance of finished goods which measure wholesale and retail-level price

n
into DTA, GST on finished goods along with changes, respectively. o
import duties on imported inputs are payable. ● Statement 1 is not correct : Wholesale Price
l.
The GST as well as the IGST paid as part of Index : It measures the changes in the prices
a

import duties will be available as credit. In case of goods sold and traded in bulk by wholesale
ri

of export of capital goods or finished goods, businesses to other businesses. Published by


the duty on imported inputs stands remitted. the Office of Economic Adviser, Ministry of
te

Also, zero rating of tax on domestic inputs is Commerce and Industry. It is the most widely
allowed. used inflation indicator in India. Major criticism
a

● Statement 2 is correct : The scheme does not for this index is that the general public does not
m

have minimum investment requirement or buy products at wholesale price. The base year
c

restriction on its location. Coupled with the of All-India WPI has been revised from 2004-05
flexibility of sourcing capital goods and inputs, to 2011-12 in 2017.
s

and flexibility for sale, the scheme is suitable ● Consumer Price Index : It measures price
p

for MSMEs.They can find most optimal changes from the perspective of a retail
.u

sourcing of inputs and capital goods and realize buyer. It is released by the National Statistical
best sale prices by selling their products in the Office (NSO). The CPI calculates the difference
w

DTA or international market, as per choice. in the price of commodities and services such
w

● Statement 3 is not correct : The scheme is as food, medical care, education, electronics
implemented by CBIC .The Central Board of etc, which Indian consumers buy for use. The
w

Indirect Taxes and Customs (CBIC), formerly CPI has several sub-groups including food and

DPP 2023 DAY 181 48


https://upscmaterial.online/
Download From - https://upscmaterial.online/

Contact us :info@onlyias.com

OnlyIAS Nothing Else Visit :dpp.onlyias.in


Contact : +91-7007 931 912

beverages, fuel and light, housing and clothing, in terms of area covered.The protected area sits
bedding and footwear. astride two major river basins of India, namely
● Statement 2 is not correct : Another the Narmada and Ganges. Three-fourths of the
difference between the two indices is that the wildlife sanctuary falls in the basin of Ganges
wholesale market is only for goods, you tributary, the Yamuna River, and one fourth of
cannot buy services on a wholesale basis. So the sanctuary falls in the Narmada basin.
WPI does not include services, whereas the
retail price index does. So WPI does not Reference:
capture changes in the prices of services, https://www.deccanherald.com/national/cheetah-
which CPI does. reintroduction-plan-in-india-optimists-sceptics-wait-
● Statement 3 is correct : Wholesale Price Index with-bated-breath-1145623.html
Categories and their respective weightages :
Primary Articles: (22.6%) Manufactured
products (64.2%)Fuel and Power (13.2%) Q.72) Ans: D
● Consumer Price Index (CPI) Categories and Exp:
their respective weightages : Food and ● Statement 1 is correct: Recently, Pakistan has
beverages (45.86%) Pan, Tobacco and begun demanding reparations, or compensation
Intoxicants (2.38%) Clothing and Footwear from the rich countries for the worst flooding
(6.53%) Housing (10%) Fuel and Light (6.84%): disaster in its history. Climate reparations refer
Electricity, LPG, Kerosene etc. (Does not to a call for money to be paid by the Global North
include Petrol and Diesel) Miscellaneous- to the Global South as a means of addressing the

e
Education, Healthcare, Transportation and historical contributions that the Global North

n
Communication etc. (28.32%) has made (and continues to make) toward

li
climate change.Countries in the Global North are
Q.71) Ans: D responsible for 92% of excess global carbon

n
Exp: emissions. Despite this, countless studies have
o
● Nauradehi sanctuary is spread over Sagar & shown that countries across the Global South are
l.
Damoh districts of Madhya Pradesh State in facing the sharpest end of the consequences
a

Central India. It came into light after visits of when it comes to climate change—from severe
ri

Wildlife Institute of India officials regarding the heat waves in India to flooding in Kenya and
Cheetah project. Noradehi sanctuary is among hurricanes in Nicaragua.The United States and
te

few wildlife sanctuaries considered an ideal the European Union, including the UK, account
location for the rehabilitation project of Cheetah for over 50% of all emissions during this time.
a

in India as its forests are not very dense to ● Statement 2 is correct: Almost the entire
m

restrict the fast movement of the fastest land developing world, particularly the small island
c

animal. And the prey base for cheetahs is also in states, has for years been insisting on setting up
abundance at the sanctuary. Nauradehi an international mechanism for financial
s

sanctuary forms a corridor between Panna Tiger compensation for loss and damage caused by
p

Reserve and Ratapani sanctuary located near climate disasters. At its heart, the demand for
.u

Bhopal.Under the Tiger Reintroduction compensation for loss and damage from
Programme, a tiger named Kishan and tigress climate disasters is an extension of the
w

Radha were brought to Nauradehi sanctuary in universally acknowledged “Polluter Pays”


w

2018 from Kanha Tiger Reserve and principle. It makes the polluter liable for paying
Bandhavgarh Tiger Reserve respectively.This is not just for the cost of remedial action, but also
w

the largest wildlife sanctuary of Madhya Pradesh for compensating the victims of environmental

DPP 2023 DAY 181 49


https://upscmaterial.online/
Download From - https://upscmaterial.online/

Contact us :info@onlyias.com

OnlyIAS Nothing Else Visit :dpp.onlyias.in


Contact : +91-7007 931 912

damage caused by their actions. The current facilities (CIF). In the case of the North Eastern
floods in Pakistan have already claimed over States and the Hilly States, the grant-in-aid will
1,300 lives, and caused economic damage worth be 90% of the common infrastructure facilities.
billions of dollars. Pakistan’s demand for ● Statement 3 is correct: The government has
reparations appears to be a long shot, but the approved two schemes, namely the scheme on
principles being invoked are fairly well- Promotion of Bulk Drug Parks and Production
established in environmental jurisprudence. Linked Incentive (PLI) Scheme to promote
domestic manufacturing of critical Key Starting
Reference: Materials/Drug Intermediates and Active
https://indianexpress.com/article/explained/explaine Pharmaceutical Ingredients in the
d-climate/the-idea-of-climate-reparation-8135293/ country.Under the PLI scheme financial incentive
will be given to eligible manufacturers of
identified 53 critical bulk drugs on their
incremental sales over the base year (2019-20)
Q.73) Ans: B for a period of 6 years. APIs, also called bulk
Exp: drugs, are significant ingredients in the
● Statement 1 is incorrect: The government aims manufacture of drugs. The Hubei province of
to develop 3 mega Bulk Drug parks through the China is the hub of the API manufacturing
Scheme for the Promotion of Bulk Drug Parks in industry.India is heavily import-dependent for
India in partnership with States. The APIs from China.The schemes are expected to
Government of India will give Grants-in-Aid to reduce manufacturing cost of bulk drugs in the

e
States with a maximum limit of Rs. 1000 Crore country and dependency on other countries for

n
per Bulk Drug Park. A sum of Rs. 3,000 crore has bulk drugs.It will also help in providing

li
been approved for this scheme for next 5 continuous supply of drugs and ensure delivery
years.Recently the Department of of affordable healthcare to the citizens.

n
Pharmaceuticals has granted ‘in-principle’ o
approval to the proposals of the three States Viz,
l.
Himachal Pradesh, Gujarat and Andhra Pradesh Q.74) Ans: D
a

under the Scheme for “Promotion of Bulk Drug Exp:


ri

Parks”. ● Statement 1 is incorrect: Market-Based


● Statement 2 is incorrect: Indian pharmaceutical Economic Dispatch (MBED) aims centralised
te

industry is the 3rd largest in the world by volume scheduling for dispatching the entire annual
and 14th largest in terms of value. India electricity consumption of around 1,400 billion
a

contributes 3.5% of total drugs and medicines units. Thus the power demand by all states is
m

exported globally. However, despite these proposed to be met through a central pool
c

achievements, India is significantly dependent allocating power at the optimal price. MBED
on the import of some of the critical basic raw aims to establish a uniform pricing framework
s

materials, viz., bulk drugs that are used to that prioritizes the least cost and most efficient
p

produce the finished dosage formulations. Three generators. The MBED model is seen as a
.u

bulk drug parks will be supported under the reduction in relative autonomy of states in
Scheme. The scheme will be implemented managing their electricity sector. The new model
w

through a State Implementing Agency (SIA), a proposes a centralised scheduling of power


w

legal entity, set up by the concerned State dispatches, both inter-state and intra-state.
Government. The grant-in-aid will be 70% of the ● Statement 2 is correct: Power is in the
w

project cost of the common infrastructure Concurrent List of the Constitution.The

DPP 2023 DAY 181 50


https://upscmaterial.online/
Download From - https://upscmaterial.online/

Contact us :info@onlyias.com

OnlyIAS Nothing Else Visit :dpp.onlyias.in


Contact : +91-7007 931 912

electricity grid is divided into state-wise ● Major Producers Globally: Thailand,


autonomous control areas managed by the State Indonesia, Malaysia, Vietnam, China
Load Dispatch Centres (SLDCs), which in turn are and India.
supervised by Regional Load Dispatch Centres ● Top Rubber Producing States: Kerala >
(RLDCs) and the National Load Dispatch Centre Tamil Nadu > Karnataka.
(NLDC).Each control area is responsible in real
time for balancing its demand with generation ● The British established the first rubber
resources.The implementation of Phase 1 of plantation in India in 1902 on the banks of the
MBED is planned to start with effect from 1st river Periyar in Kerala. India is currently the
April 2022. Before this CERC will align their fifth largest producer of Natural Rubber in the
regulations and mock drills will be carried out to world with one of the highest productivity. The
ensure that the system runs smoothly. Rubber Plantation Development Scheme and
● Statement 3 is correct: MBED as a way forward the Rubber Group Planting Scheme are
to deepen power markets in line with the examples of government-led initiatives for
Centre’s ‘One Nation, One Grid, One Frequency, rubber. 100% Foreign Direct Investment (FDI)
One Price’ formula. Synchronisation of all is allowed in plantations of rubber.
regional grids will help in optimal utilization of Reference:
scarce natural resources by transfer of Power https://www.google.com/amp/s/indianexpress.com/a
from Resource centric regions to Load centric rticle/cities/pune/rubber-industry-government-help-
regions.Maintaining a consistent electrical crisis-8100943/lite/
frequency is important because multiple

e
frequencies cannot operate alongside each

n
other without damaging equipment. Q.76) Ans: B

li
Exp:
● The Direct Benefit Transfer (DBT) scheme

n
came into effect on 1 January 2013. It was
o
Q.75) Ans: C launched to enhance the Indian Government's
l.
Exp: delivery mechanism and reform existing
a

● Commercial Plantation Crop: welfare scheme procedures. The DBT scheme


ri

Rubber is made from the latex of a tree called simplifies the transfer of benefits, such as
Hevea Brasiliensis. Rubber is largely perceived subsidies, to the beneficiaries, making the
te

as a strategic industrial raw material and process smooth, secure, and hassle-free. The
accorded special status globally for defence, DBT scheme helps the Government directly
a

national security and industrial development. deposit subsidies into the beneficiaries'
m

Conditions for Growth: It is an equatorial crop, accounts.


c

but under special conditions, it is also grown in The following are the key benefits of the DBT
tropical and sub-tropical areas. scheme:
s

Temperature: Above 25°C with a moist and ➢ The DBT scheme prevents fraud. The
p

humid climate. Government sends the funds straight to the


.u

Rainfall: More than 200 cm. beneficiaries’ account, which removes the
Soil Type: Rich well-drained alluvial soil.
w

possibility of fraud through a middleman.


Cheap and adequate supply of skilled labour is ➢ The beneficiaries can be recognised with the
w

needed for this plantation crop. use of their Aadhaar number. Because Aadhaar
is a universal ID, the Government can easily
w

DPP 2023 DAY 181 51


https://upscmaterial.online/
Download From - https://upscmaterial.online/

Contact us :info@onlyias.com

OnlyIAS Nothing Else Visit :dpp.onlyias.in


Contact : +91-7007 931 912

verify the beneficiaries using their Aadhaar (Prevention, Care and Rehabilitation) Bill, 2021.
details. This Bill aims to tackle all aspects of trafficking
➢ DBT promotes accountability in subsidy including the social and economic causes of the
distribution. As a result, it aids in the crime, punishment to traffickers, and the
elimination of inconsistency and delay in protection and rehabilitation of survivors.
payments. ● Statement 2 is incorrect: India has ratified the
➢ DBT aids in the distribution of subsidies to United Nations Convention on Transnational
deserving applicants living below the poverty Organised Crime (UNCTOC) which among others
level. It helps the Government reach out to the has a Protocol to Prevent, Suppress and Punish
intended beneficiaries with ease. Trafficking in Persons, especially Women and
Children.It is also known as the Palermo
➢ The scheme terminates pilferage in the
Convention. The United Nations Office on Drugs
distribution of money and reduces the misuse
and Crime (UNODC) is the custodian of the
of public funds.
UNTOC. India has ratified the SAARC
➢ DBT is a powerful transaction and settlement Convention on Preventing and Combating
technology that works with multiple
Trafficking in Women and Children for
organisations.
Prostitution.
➢ DBT has proven to be an effective technique
for connecting with people to distribute relief
funds.
➢ DBT helps in saving time and cost by Q.78) Ans: C

e
eliminating intermediaries. Exp:
➢ DBT

n
Increasing transparency through ● The Central Bureau of Investigation (CBI)

li
eliminating ghost beneficiaries. conducted searches at 59 locations across 20
States and one Union Territory, as part of a

n
pan-India drive against the circulation and
o
Q.77) Ans: C sharing of child sexual abuse material.The
l.
Exp: operation code-named “Megh Chakra” was
a

● Statement 1 is incorrect: Trafficking in Human carried out following the inputs received from
ri

Beings or Persons is prohibited under the the Interpol’s Singapore special unit based on
Constitution of India under Article 23 (1). The the information received from the authorities
te

Immoral Traffic (Prevention) Act, 1956 (ITPA) is in New Zealand.The operation is targeted at
the premier legislation for the prevention of cloud storage facilities used by the peddlers
a

human trafficking for commercial sexual to circulate audio-visuals of illicit sexual


m

exploitation. Criminal Law (Amendment) Act activities with minors, thus earning the code
2013 has come into force wherein Section 370 of
c

‘Megha Chakra’.After the Supreme Court’s


the Indian Penal Code has been substituted with order, the Department of Telecommunication
s

Section 370 and 370A IPC which provide for banned several websites containing child
p

comprehensive measures to counter the pornographic material.As per the Information


.u

menace of human trafficking including the Technology (IT) Act, 2002, it is punishable to
trafficking of children for exploitation in any show children any pornographic content.In
w

form including physical exploitation or any form 2021 alone, more than 29 million reports of
w

of sexual exploitation, slavery, servitude, or the suspected child sexual exploitation were
forced removal of organs.The Government of reported by online platforms to the National
w

India has proposed the Trafficking in Persons Center for Missing & Exploited Children’s

DPP 2023 DAY 181 52


https://upscmaterial.online/
Download From - https://upscmaterial.online/

Contact us :info@onlyias.com

OnlyIAS Nothing Else Visit :dpp.onlyias.in


Contact : +91-7007 931 912

CyberTipline.In a global compilation of reports Exp:


of Child Sexual Abuse Material (CSAM) found ● Statement 1 is incorrect: Section 437, Criminal
online, India stands on top of the list, with Procedure Code, deals with the powers of the
11.7% of the total reports followed by trial court and of the Magistrate to whom the
Pakistan, which contributes 6.8% of all reports. offender is produced by the police or the
accused surrenders or appears, to grant or
refuse bail to person accused of,or suspected of
Q.79) Ans: D the commission of any non-bailable offence.
Exp: Section 437 of the CrPC deals with bail in case
● Statement 1 is incorrect: National Legal Services of non-bailable offences. It says a person shall
Authority (NALSA) Constituted under the Legal not be released on bail if there is reasonable
Services Authorities Act, 1987. Its Objective is to ground to believe that he has committed an
provide free Legal Services to the weaker offence punishable with death or life
sections of the society and to organize Lok imprisonment; or, if he has been previously been
Adalats for amicable settlement of disputes. convicted for an offence punishable with death,
Article 39A of the Constitution of India provides life imprisonment, or for a term of seven years
that State shall secure that the operation of the or more; or been convicted on two or more
legal system promotes justice on a basis of equal occasions on other offences with a term
opportunity, and shall in particular, provide free between three and seven years.
legal aid, by suitable legislation or schemes or in ● Statement 2 is correct: However, Section 437 of
any other way, to ensure that opportunities for the CrPC also contains exceptions in a proviso

e
securing justice are not denied to any citizen by that says the court may grant bail even in these

n
reason of economic or other disability. Articles cases, “if such person is under the age of 16 or is

li
14 and 22(1) also make it obligatory for the State a woman or is sick or infirm”. Recently the
to ensure equality before law and a legal system Supreme Court allowed activist Teesta Setalvad

n
which promotes justice on a basis of equal interim bail in a case of alleged forgery and
o
opportunity to all. fabrication of records in connection with the
l.
● Statement 2 is incorrect: While the CJI is the 2002 Gujarat riots cases.A three-judge Special
a

Patron-in-Chief, the second senior most judge Bench led by Chief Justice of India said a case of
ri

of the Supreme Court of India is the executive interim bail is “evidently made out” on
chairperson of the NALSA. In every State, the consideration of the facts that she is a woman
te

State Legal Services Authority has been who has already undergone seven days of
constituted to give effect to the policies and custodial interrogation. CJI made an oral
a

directions of the NALSA and to give free legal observation that under Section 437 of the Code
m

services to the people and conduct Lok Adalats of Criminal Procedure (CrPC), a woman is
c

in the State.The State Legal Services Authority is entitled to favourable treatment.


headed by Hon’ble the Chief Justice of the
s

respective High Court. Recently, Justice


p

Chandrachud took up the position that became Q.81) Ans: C


.u

vacant after Justice U.U. Lalit was elevated as the Exp:


49th Chief Justice of India (CJI). ● The doctrine of “essentiality” was invented
w

by a seven-judge Bench of the Supreme


w

Court in the ‘Shirur Mutt’ case in 1954. The


court held that the term “religion” will cover
w

Q.80) Ans: A all rituals and practices “integral” to a

DPP 2023 DAY 181 53


https://upscmaterial.online/
Download From - https://upscmaterial.online/

Contact us :info@onlyias.com

OnlyIAS Nothing Else Visit :dpp.onlyias.in


Contact : +91-7007 931 912

religion, and took upon itself the carved from a monolithic granite stone and
responsibility of determining the essential weighs 65 MT.
and non-essential practices of a
religion.Essential religious practice test is a ● Statement 2 is incorrect: Subhas Chandra Bose
contentious doctrine evolved by the court to was born on 23rd January 1897, in Cuttack,
protect only such religious practices which Orissa Division, Bengal Province, to Prabhavati
were essential and integral to the religion. In Dutt Bose and Janakinath Bose. His Jayanti is
the legal framework, the doctrine of celebrated as 'Parakram Diwas' on 23rd
essentiality is a doctrine that has evolved to January. In 1919, he had cleared the Indian
protect the religious practices that are Civil Services (ICS) examination. Bose,
essential or integral and does not violate however, resigned later.He was highly
any fundamental right. India being a secular influenced by Vivekananda's teachings and
country has discrete religious beliefs and to considered him as his spiritual Guru. His
deny any is to violate the freedom of religion. political mentor was Chittaranjan Das. He
Article 25 guarantees the freedom of worked as the editor for Das’s newspaper–
conscience, the freedom to profess, Forward, and later started his own newspaper,
practice, and propagate religion to all Swaraj.
citizens. The above-mentioned freedoms are 1. Satyendranath Tagore was the first Indian to
subject to public order, health, and morality. join the Indian Civil Service in the year 1864.
Recently A two-judge Bench of the Supreme 2. In July 1920, Bose took the ICS exams in
Court of India is presently hearing arguments London and came fourth. Bose then faced a

e
on the correctness of a Karnataka High Court dilemma as to whether to take up this

n
judgement that upheld the ban on the use of opportunity and sought advice from his family

li
the hijab by students in Karnataka.The through correspondence to India. Finally in
petitioners claim that the judgment of the April 1921, Bose withdrew from taking up this

n
Karnataka High Court is violative of the rights post with the ICS and returned to India in the
o
of students to freedom of expression, summer of 1921.
l.
conscience, and religion and would also
a

adversely impact the right to education of ● Statement 3 is incorrect:


ri

Muslim women. He reached Japanese-controlled Singapore


from Germany in July 1943, issued from there
te

his famous call, ‘Delhi Chalo’, and announced


Q.82) Ans: D the formation of the Azad Hind Government
a

Exp: and the Indian National Army on 21st October


m

● Statement 1 is correct: Kartavya Path 1943.


symbolises a shift from erstwhile Rajpath being
c

The Indian National Army was first formed


an icon of power to Kartavya Path being an under Mohan Singh and Japanese Major
s

example of public ownership and Iwaichi Fujiwara and comprised Indian


p

empowerment. The statue of Netaji Subhas prisoners of war of the British-Indian Army
.u

Chandra Bose made of granite, is a fitting captured by Japan in the Malayan (present-day
tribute to the immense contribution of Netaji Malaysia) campaign and at Singapore. The INA
w

to our freedom struggle, and would be a included both the Indian prisoners of war from
w

symbol of the country’s indebtedness to him. Singapore and Indian civilians in South-East
Crafted by Shri Arun Yogiraj, who was the main Asia. Its strength grew to 50,000. The INA
w

sculptor, the 28 feet tall statue has been fought allied forces in 1944 inside the borders

DPP 2023 DAY 181 54


https://upscmaterial.online/
Download From - https://upscmaterial.online/

Contact us :info@onlyias.com

OnlyIAS Nothing Else Visit :dpp.onlyias.in


Contact : +91-7007 931 912

of India in Imphal and in Burma. In November The UNCLOS led to the establishment of
1945, a British move to put the INA men on territorial sea boundaries 22 km offshore,
trial immediately sparked massive deciding the region up to which countries
demonstrations all over the country. could claim full sovereign territorial rights, as
well as the 200 nautical miles EEZ limit. It also
created the International Seabed Authority
and other conflict-resolution mechanisms.
Q.83) Ans: C
Exp:
● Statement 1 is correct: UN High Seas Treaty Q.84)
Also referred to as the ‘Paris Agreement for the Ans: D
Ocean’, the treaty to deal with Biodiversity Exp:
Beyond National Jurisdiction has been under ● Statement 1 is incorrect: Ramappa Temple,
discussion for several years.The proposed also known as the Rudreshswara (Lord Siva)
treaty concerns the ocean existing beyond the temple, is a UNESCO World Heritage Site
Exclusive Economic Zones that lie from the located in the state of Telangana. It lies in a
coast of a country to about 200 nautical miles valley in Palampet village of Venkatapur
or 370 km into the sea, till where it has special Mandal of Mulugu district, a tiny village long
rights for exploration. Waters beyond that are past its days of glory in the 13th and 14th
known as open seas or high seas. centuries.

e
● Statement 2 is incorrect: The UN High Sea ● Statement 2 is incorrect: The temple

n
Treaty was to be negotiated under the United complexes of Kakatiyas have a distinct style,

li
Nations Convention on Laws of the Sea technology, and decoration exhibiting the
(UNCLOS) of 1982 which governs the rights of influence of the Kakatiyan sculptor.

n
countries regarding marine resources. As there The temple stands on a 6 feet high star-shaped
o
is no treaty for conserving the health of vast platform with walls, pillars, and
l.
swathes of the earth’s oceans, a UN resolution ceilings adorned with intricate carvings that
a

in 2017 had decided to rectify this while setting attest to the unique skill of the Kakatiyan
ri

2022 as the deadline.It doesn't exclude sculptors.The foundation is built with the
UNCLOS. “sandbox technique”, the flooring is granite,
te

and the pillars are basalt. The Kakatiya


● Statement 3 is correct : Some aspects of temples, dedicated mostly to Shiva, reveal in
a

negotiations included establishing marine their construction a happy blending of the


m

protected areas to put limits on certain styles of North India and South India which
c

activities, environmental impact assessments influenced the political life of the Deccan.They
or clearances for sustainability of works, adopted both the North Indian Nagara Bhumija
s

financial support to countries and sharing style and the South Indian Dravidian style.
p

other scientific knowledge. The International The most important of these temples are those
.u

Union for Conservation of Nature has said at Palampet (Ramappa temple),


binding agreements are needed for this treaty Hanamkonda (Thousand Pillared temple) and
w

to be effective. the temples in the Warangal fort including the


w

big ruined temple complex —


Extra Info: Swayambhunadha temple
w

DPP 2023 DAY 181 55


https://upscmaterial.online/
Download From - https://upscmaterial.online/

Contact us :info@onlyias.com

OnlyIAS Nothing Else Visit :dpp.onlyias.in


Contact : +91-7007 931 912

● Statement 3 is correct: An inscription in the diarrhea, while others cause urinary tract
temple dates it to the year 1213 CE and says it infections, respiratory illness and pneumonia,
was built by a Kakatiya General Recharla Rudra and other illnesses. It is a common pathogen
Deva, during the period of the Kakatiya ruler in ICUs worldwide.Usually, it is harmless and
Ganapati Deva. This temple was constructed quite beneficial for the health of the organism.
by Kakatiya king Ganapati Deva’s general But some variations of these bacteria are quite
Recharla harmful and can cause severe food poisoning.
Rudra. Besides food poisoning, E.coli is also known to
cause pneumonia as well as urinary tract
infections, the latter of which can lead to
serious health complications. One of the
primary reasons why people fall sick is because
Q.85) Ans: D certain strains of E.coli bacteria produce a
Exp: potent toxin called Shiga. The Shiga toxin can
● Statement 1 is not correct : E. coli are a large damage the lining of the intestine, which can
and diverse group of bacteria that are found eventually lead to a host of other illnesses.
in environment, foods, and intestines of Consequently, the particular strains of E.coli
people and animals. Escherichia coli (E. coli) is that produce this toxin are called Shiga Toxin-
a bacteria that is commonly found in the producing E.coli (abbreviated as STEC).
lower intestine of warm-blooded organisms.
E.coli or Escherichia coli is a rod-shaped

e
bacteria that generally lives in the intestines

n
of humans and most other mammals. Usually, Q.86) Ans: C

li
it is harmless and quite beneficial for the Exp:
health of the organism. But some variations of ● Option 3 is correct : Einstein ring : In

n
these bacteria are quite harmful and can cause observational astronomy, an Einstein ring
o
severe food poisoning. Shiga toxin-producing (Einstein-Chwolson ring) is the deformation
l.
E. coli (STEC) is a bacterium that can cause (distortion) of the light from a source (such as
a

severe foodborne disease. Primary sources of a star or galaxy) into a ring through
ri

STEC outbreaks are raw or undercooked gravitational lensing of the source’s light by
ground meat products, raw milk, and faecal an object with an extremely large mass (such
te

contamination of vegetables. In most cases, as a black hole or another galaxy). The


the illness is self-limiting, but it may lead to a distortion is produced by the bending of the
a

life-threatening disease including haemolytic light rays from the source due to a massive
m

uraemic syndrome (HUS), especially in young galaxy is termed the It lies between source and
c

children and the elderly. STEC is heat-sensitive. the observer. The strong gravitational field
In preparing food at home, be sure to follow produced by the lens galaxy distorts the
s

basic food hygiene practices such as "cook structure of space-time in its neighbourhood.
p

thoroughly". Following the WHO “Five keys to It attracts objects which have a mass and
.u

safer food” is a key measure to prevent bends paths of light. When the two galaxies are
infections with foodborne pathogens such as exactly aligned, the image of the more distant
w

STEC. galaxy is converted into an almost perfect


w

● Statement 2 is not correct : Although most circle which surrounds the lens galaxy. The
strains of E. coli are harmless, others can irregularities in the circle are due to
w

make you sick. Some kinds of E. coli can cause asymmetries in the source galaxy.NASA's

DPP 2023 DAY 181 56


https://upscmaterial.online/
Download From - https://upscmaterial.online/

Contact us :info@onlyias.com

OnlyIAS Nothing Else Visit :dpp.onlyias.in


Contact : +91-7007 931 912

James Webb Space Telescope (JWST) snapped designation, International Dark Sky Sanctuary,
a perfect shot of an 'Einstein Ring'. Launched was introduced in 2015.
in 2021, JWST is NASA’s largest and most ● Statement 2 is not correct : Department of
powerful space telescope to probe cosmos to Science & Technology has announced setting
uncover history of universe. Einstein Ring is up of India’s first dark sky reserve at Hanle in
when light from a star or a galaxy passes Ladakh as a part of Changthang Wildlife
another galaxy or a massive object on its way Sanctuary. It will be one of the world’s highest-
towards Earth. Since gravitational force of located sites for optical, infra-red, and gamma-
large object bends the light, it develops ray telescopes. It will promote astronomy-
gravitational lensing, forming a ring-like effect, tourism, giving a boost to local tourism and
making the light of galaxy appear as an almost economy through science.
perfect ring ● Statement 3 is not correct : Cold Desert with
Sparse Population: The Indian Astronomical
Q.87) Ans: A Observatory, the high-altitude station of IIA, is
Exp: situated to the north of the Western
● Statement 1 is correct : A dark-sky reserve is Himalayas, at an altitude of 4,500 metres
an area, usually surrounding a park or above mean sea level. Located atop Mt.
observatory that is kept free of artificial light Saraswati in the Nilamkhul Plain in the Hanle
pollution. The purpose of a dark sky preserve is Valley of Changthang, it is a dry, cold desert
generally to promote astronomy. Dark Sky with a sparse human population and has the
Reserve is a place that has policies to ensure Hanle monastery as its nearest neighbour.

e
that a tract of land or region has minimal Hanle in Ladakh with Cloudless skies and low

n
artificial light interference. International Dark atmospheric water vapour make it one of the

li
Sky Association, a U.S.-based non-profit, best sites in world for astronomical
designates places as International Dark Sky observations. The cloudless skies and low

n
Places, parks, sanctuaries and reserves, atmospheric water vapour make it one of the
o
depending on criteria they meet.A dark-sky best sites in the world for optical, infrared,
l.
preserve (DSP) is an area, usually surrounding sub-millimetre, and millimetre wavelengths.
a

a park or observatory, that restricts artificial Other Telescopes situated in Hanle


ri

light pollution. The purpose of the dark-sky Observatory: The Himalayan Chandra
movement is generally to promote astronomy. Telescope (HCT), High Energy Gamma Ray
te

However, astronomy is certainly not the only telescope (HAGAR), the Major Atmospheric
objective of conserving a dark sky. A dark night Cherenkov Experiment Telescope (MACE) and
a

sky is associated with many facets of history, GROWTH-India are prominent telescopes
m

philosophy, religion, societal development, located at the Hanle observatory.


c

poetry, song, mathematics, and science.


Hence, dark sky preservation is necessary to
s

understand our environmental history. Q.88) Ans: A


p

Different terms have been used to describe the Exp:


.u

areas as national organizations have worked ● Statement 1 is incorrect: Shrink inflation is


independently to create their programs. The when a product downsizes its quantity while
w

International Dark-Sky Association (IDA) uses keeping the price the same. Downsizing of
w

International Dark Sky Reserve (IDSR) and products is done to offset higher production
International Dark Sky Park (IDSP). A third costs but keep retail prices the same. Price
w

points become misleading when the basket of

DPP 2023 DAY 181 57


https://upscmaterial.online/
Download From - https://upscmaterial.online/

Contact us :info@onlyias.com

OnlyIAS Nothing Else Visit :dpp.onlyias.in


Contact : +91-7007 931 912

goods cannot always be measured by in India and mobilise international capital


considering the product size. Overall inflation markets to support infrastructure development
must be tackled to address shrinkflation. It can in the country. In 2019, Kerala became the first
be done with a mix of macroeconomic policies to Indian state to issue Masala Bonds worth Rs.
manage demand and supply and address 2,150 crore on the London Stock Exchange.
structural rigidities in the economy. State-owned Kerala Infrastructure Investment
● Statement 2 is correct: An increase in the cost Fund Board (KIIFB) had issued the bonds to raise
of input materials and intense competition funds in the overseas market.
results in Shrinkflation. Inflation has numerous ● Statement 3 is correct: Any corporate and
effects on the daily life of consumers such as on Indian bank is eligible to issue Rupee
rent, food, gas, and other living expenses. Since denominated bonds overseas.Money raised
container and vessel sizes are reduced by through these bonds cannot be invested in real-
extremely minor quantities, shrinkflation tricks estate activities. However, they can be used for
consumers into thinking that the brands they the development of integrated township or
purchase are unaffected by inflation. affordable housing projects.Also, the money
Shrinkflation can occur in different ways other raised through Masala Bonds cannot be invested
than changes in quantity, by reformulating or in capital markets.These bonds can only be
removing ingredients while maintaining its price issued to a resident of such a country which is a
to keep consumers from switching to different member of the Financial Action Task Force
brands. Shrinkflation can lead to customer (FATF). Also, the security market regulator of the
frustration and deterioration of consumer country must be a member of the International

e
sentiment towards a producer’s brand. Organisation of Securities Commission.These

n
bonds can also be subscribed by regional and

li
multilateral financial institutions where India is a
member country.

n
Q.89) Ans: C o
Exp:
l.
● Statement 1 is incorrect: Masala Bonds are
a

rupee-denominated bonds issued outside India Q.90) Ans: B


ri

by Indian entities. They are debt instruments Exp:


which help to raise money in local currency ● Statement 1 is incorrect: CRISPR is short for
te

from foreign investors. Both the government Clustered Regularly Interspaced Short
and private entities can issue these bonds.The Palindromic Repeats, which is a reference to the
a

major objectives of Masala Bonds are to fund clustered and repetitive sequences of DNA
m

infrastructure projects, ignite internal growth found in bacteria, whose natural mechanism to
c

(via borrowings) and internationalise the Indian fight some viral diseases is replicated in this
rupee. In case of any risk, the investor has to gene-editing tool.CRISPR technology does not
s

bear the loss and not the borrower. involve the introduction of any new gene from
p

● Statement 2 is correct: The first Masala bond the outside.CRISPR-Cas9 technology is often
.u

was issued in 2014 by International Finance described as ‘Genetic Scissors’.A vast number of
Corporation (IFC) for the infrastructure projects diseases and disorders are genetic in nature i.e.;
w

in India.The International Finance Corporation they are caused by unwanted changes or


w

(IFC), the investment branch of the World Bank mutations in genes.CRISPR presents a potential
issued a 10-year, 10 billion Indian rupee bonds in treatment for the cure of such diseases and
w

November 2014 to increase foreign investment disorders .

DPP 2023 DAY 181 58


https://upscmaterial.online/
Download From - https://upscmaterial.online/

Contact us :info@onlyias.com

OnlyIAS Nothing Else Visit :dpp.onlyias.in


Contact : +91-7007 931 912

● Statement 2 is correct: CRISPR technology Pacific Ocean, which can cause droughts, fierce
mechanism is often compared to the ‘cut-copy- winds and heavy rainfall.
paste’, or ‘find-replace’ functionalities in According to WMO, the current La Nina is
common computer programmes. A bad stretch projected to span three consecutive northern
in the DNA sequence, which is the cause of hemisphere winters. It began in September
disease or disorder, is located, cut, and removed 2020.
— and then replaced with a ‘correct’ sequence. If it continues for the next six months, it will be
And the tools used to achieve this are not the first “triple-dip” La Nina event of the 21st
mechanical, but biochemical — specific protein century, WMO says.
and RNA molecules.The technology replicates a
natural defence mechanism in some bacteria ● Statement 2 is correct: In the Indian context,
that uses a similar method to protect itself from La Nina is associated with good rainfall during
virus attacks. the monsoon season. This is the opposite of El
● Statement 3 is correct: India has approved a 5- Nino which is known to suppress monsoon
year project to develop CRISPR to cure sickle rainfall. Thus, a continued spell of La Nina
cell anaemia.Sickle cell anaemia is the first could lead to expectation of another year of
disease that is being targeted for CRISPR-based good, or normal, rainfall during the monsoon.
therapy in India. Sickle cell anaemia is an
inherited blood disease which is most common ● Statement 3 is incorrect: The Indian Ocean
among people of African, Arabian and Indian Dipole (IOD) is defined by the difference in sea
origin.It is a group of disorders that affects surface temperature between two areas (or

e
haemoglobin, the molecule in red blood cells poles, hence a dipole) – a western pole in the

n
that delivers oxygen to cells throughout the Arabian Sea (western Indian Ocean) and an

li
body.People with this disease have atypical eastern pole in the eastern Indian Ocean
haemoglobin molecules called haemoglobin S, south of Indonesia. The IOD affects the climate

n
which can distort red blood cells into a sickle, or of Australia and other countries that surround
o
crescent shape.This blocks blood flow and the Indian Ocean Basin, and is a significant
l.
oxygen from reaching all parts of the body. contributor to rainfall variability in this region.
a
ri

Positive event:
te

Q.91) Ans: C 1. Warmer sea surface temperatures in the


Exp: western Indian Ocean relative to the east.
a

● Statement 1 is incorrect: La Nina refers to the 2. Easterly wind anomalies across the Indian
m

large-scale cooling of the ocean surface Ocean and less cloudiness to Australia's
c

temperatures in the central and eastern northwest


equatorial Pacific Ocean, coupled with changes 3. Less rainfall over southern Australia and the
s

in the tropical atmospheric circulation, namely Top End.


p

winds, pressure and rainfall. It usually has the


.u

opposite impacts on weather and climate as El Negative event:


Nino, which is the warm phase of the so-called
w

El Niño Southern Oscillation (ENSO). 1. Cooler sea surface temperatures in the


w

A “triple-dip” La Nina is a multiyear cooling of western Indian Ocean relative to the east
the surface temperature of the equatorial 2. Winds become more westerly, bringing
w

increased cloudiness to Australia's northwest

DPP 2023 DAY 181 59


https://upscmaterial.online/
Download From - https://upscmaterial.online/

Contact us :info@onlyias.com

OnlyIAS Nothing Else Visit :dpp.onlyias.in


Contact : +91-7007 931 912

3. More rainfall in the Top End and southern branch of the Barak River when the Barak
Australia. separates into the Kushiyara and Surma.
The waters of the Kushiyara originate in the
Positive IOD phase: state of Nagaland and pick up tributaries from
Westerly winds weaken along the equator Manipur, Mizoram and Assam.
allowing warm water to shift towards Africa.
Changes in the winds also allow cool water to
rise up from the deep ocean in the east. This
sets up a temperature difference across the
tropical Indian Ocean with cooler than normal
water in the east and warmer than normal
water in the west. Generally this means there
is more moisture than normal in the
atmosphere over the West Indian Ocean &
Arabian Sea.
This changes the path of weather systems Q.93) Ans: D
coming towards India, often resulting in more Exp:
rainfall during Southwest Monsoon. ● The Kazakhstan–Kyrgyzstan border is 1,212
kilometres (753 mi) and runs from the tripoint
with Uzbekistan to the tripoint with China.
Q.92) Ans: B Bishkek, the Kyrgyz capital, is situated just 16

e
Exp: km to the south of this boundary, and Almaty
(Kazakhstan's largest city and former capital) is

n
● Statement 1 is incorrect: Kushiyara River has
its origin at the mouth of the Barak, also known situated just 29 kilometres to the north of it.

li
as the Amshid bifurcation point. The Kushiyara

n
flows westward forming the boundary o
between Assam, India, and the Sylhet District
l.
of Bangladesh.
a

It flows between the towns of Zakiganj, Sylhet,


ri

and Karimganj, Assam, and after the village of Both countries are bordered by China in its
Panjipuri enters entirely into the Beanibazar South - East while Uzbekistan in its South by
te

Upazila of Bangladesh. Kazakhstan and South- West by Kyrgyzstan.


Hence both Brahmaputra and Kushiyara River
a

doesn't originate from Chemayungdung Reference:


m

Glacier. https://www.google.com/amp/s/indianexpress.com/a
The Brahmaputra's source is the
c

rticle/world/kyrgyz-tajik-border-conflict-escalates-
Chemayungdung Glacier, which covers the with-use-of-heavy-weaponry-8154437/lite/
s

slopes of the Himalayas about 60 miles (100


p

km) southeast of Lake Mapam in southwestern


.u

Tibet. Q.94) Ans: C


Exp:
w

● Statement 2 is correct: Kushiyara River is a ● Statement 1 is correct: Indian Computer


w

distributary river in Bangladesh and Assam. It Emergency Response Team (CERT-In) under
forms on the India-Bangladesh border as a Ministry of Electronics & IT, Government of
w

India, successfully designed & conducted the

DPP 2023 DAY 181 60


https://upscmaterial.online/
Download From - https://upscmaterial.online/

Contact us :info@onlyias.com

OnlyIAS Nothing Else Visit :dpp.onlyias.in


Contact : +91-7007 931 912

Cyber Security Exercise "Synergy" as part of the 4. Coordination of cyber incident response
International Counter Ransomware Initiative- activities.
Resilience Working Group which is being led by 5. Issue guidelines, advisories, vulnerability notes
India under the leadership of National Security and whitepapers relating to information
Council Secretariat (NSCS). security practices, procedures, prevention,
response and reporting of cyber incidents.
● Statement 2 is incorrect: India is among the
four countries that have volunteered to
organise and lead specific thematic discussions Q.95) Ans: B
at the first-of-its-kind international meeting to Exp:
counter ransomware convened by the USA in ● The Defence Acquisition Council (DAC) has
October 2021 that will chalk out a four-part approved the issuance of a Request For
strategy to address the challenges posed in the Proposal (RFP) for the construction of six
cyberworld. conventional submarines under Project-75I
Four-Part Strategy Four countries have (India).RFP is a project announcement posted
volunteered to lead and organise specific publicly by an organisation indicating that bids
thematic discussions as part of the USA lead for contractors to complete the project are
International Counter Ransomware Initiative: sought.
1. India for resilience (bolstering resilience to About the Project:
withstand ransomware attacks) This project envisages indigenous construction
2. Australia for disruption (disrupt ransomware of submarines equipped with the state-of-the-

e
infrastructure and actors) art Air Independent Propulsion system at an

n
3. UK for virtual currency (abuse of virtual estimated cost of Rs. 43,000 crore.Project 75

li
currency to launder ransom payments) (I), approved in 2007, is part of the Indian
4. Germany for diplomacy (leveraging Navy’s 30 year Plan for indigenous submarine

n
international cooperation to disrupt the construction.It will be the first under the
o
ransomware ecosystem and address safe strategic partnership model which was
l.
harbours for ransomware criminals). promulgated in 2017 to boost indigenous
a

defence manufacturing.
ri

● Statement 3 is correct: Indian Computer


Emergency Response Team (CERT-In) CERT-In,
te

under Ministry of Electronics & IT, is the Q.96) Ans: C


national nodal agency for responding to Exp:
a

computer security incidents as and when they ● Statement 1 is incorrect: Atal bridge is built on
m

occur. Under the Information Technology the banks of the Sabarmati Riverfront,
c

Amendment Act 2008, CERT-In has been Sabarmati River which flows through the city of
designated to serve as the national agency to Ahmedabad.
s

perform the following functions in the area of This bridge is about 300 meters long and 14
p

cyber security: meters wide.


.u

Sabarmati River is a monsoon-fed river


1. Collection, analysis and dissemination of originating in Aravalli hills (Udaipur) and
w

information on cyber incidents. draining in the Gulf of Cambay in the Arabian


w

2. Forecast and alerts of cyber security incidents Sea.


3. Emergency measures for handling cyber Hence statement 1 is incorrect.
w

security incidents

DPP 2023 DAY 181 61


https://upscmaterial.online/
Download From - https://upscmaterial.online/

Contact us :info@onlyias.com

OnlyIAS Nothing Else Visit :dpp.onlyias.in


Contact : +91-7007 931 912

● Statement 2 is incorrect: increasing its expenditure, private


Sabarmati River has Luni at its North and expenditure comes down.Higher
Narmada at its South. borrowing pushes up market interest
However Luni is a river in Rajasthan. It rate. Increased government borrowing
originates in the Pushkar valley of the Aravalli in the market compels the private
Range, near Ajmer, passes through the sector to compete with the
southeastern portion of the Thar Desert, and government for money. As a result,
ends in the marshy lands of Rann of Kutch in market interest rate goes up.This is for
Gujarat, after travelling a distance of 495 km. It two main reasons.With expansionary
doesn't drain in the Arabian sea. fiscal policy, private sector savers buy
government bonds and so have fewer
The Narmada, the largest west flowing river of savings to fund private sector
the Peninsula, rises near the Amarkantak range investment. Also, higher government
of mountains in Madhya Pradesh. It is the fifth borrowing tends to push up interest
largest river in the country and the largest one rates and these higher interest rates
in Gujarat. It traverses Madhya Pradesh, reduce investment. Increased interest
Maharashtra and Gujarat and meets the Gulf rate and reduced availability of funds
of Cambay in the Arabian sea. discourages private spending. Or in
other words, high government
expenditure crowds out private
expenditure. A high magnitude of the

e
crowding-out effect may even lead to

n
lesser income in the economy due to

li
low investment in the economy by
● Statement 3 is correct: the private sector.

n
Sabarmati river originates from Aravalli ➢ Crowding in – The crowding in effects
o
Hills,Rajasthan and meets in Bay of Khambhat occurs because higher government
l.
in Arabian sea. Its length is 371 km. & total spending leads to an increase in
a

catchment area is 21,674 sq. km. economic growth and therefore


ri

Sei, Siri and Dhamni are the right bank’s encourages firms to invest because
tributaries and Wakal, Harnav, Hathmati, there are now more profitable
te

Khari, Watrak are left bank’s tributaries. investment opportunities.This relates


a

to how higher government spending


encourages firms to invest more.This is
m

Q.97) Ans: B due to the income effect of higher


Exp:
c

government spending. If the economy


● When the government pursues expansionary is in a recession or below full capacity,
s

fiscal policy (higher spending financed by expansionary fiscal policy can increase
p

borrowing) there are two possible effects: the economic growth rate and create
.u

➢ Crowding out – The crowding out a positive multiplier effect, which


effect refers to a situation of high
w

leads to greater private sector


government expenditure supported by investment.
w

high borrowing causes decrease in


private expenditure. Or in other Q.98) Ans: D
w

words, when the government is


Exp:
DPP 2023 DAY 181 62
https://upscmaterial.online/
Download From - https://upscmaterial.online/

Contact us :info@onlyias.com

OnlyIAS Nothing Else Visit :dpp.onlyias.in


Contact : +91-7007 931 912

● Statement 1 is correct: The United Nations ● Statement 1 is incorrect: Coral reefs are formed
Environmental Programme(UNEP), along with when the freely moving larvae of corals attach
UK-based charity Reef-World Foundation has themselves to sedimentary rocks or hard
launched the Green Fins Hub. The Green Fins surfaces near the coastlines. This process is
Hub is the first-ever global marine tourism undertaken with the help of several other
industry platform.The hub offers enhanced and processes like sedimentation, compaction,
global membership for participating dive and cementation and solidification of the skeletons
snorkel operators to conserve the marine of coral polyps.The majority of reef-building
environment. The platform will help diving and corals are found within tropical and subtropical
snorkelling operators worldwide to make simple, waters. These typically occur between 30°
cost-efficient changes to their daily practices by north and 30° south latitudes.The
utilizing tried and tested solutions.It would also Indonesian/Philippines archipelago has the
help them keep track of their annual world’s greatest concentration of reefs and the
improvements and communicate with their greatest coral diversity.Other areas of reef
communities and customers. concentration are the Great Barrier Reef of
● Statement 2 is correct: Green Fins is a proven Australia, the Red Sea, and the Caribbean, the
conservation management approach – latter having a much lower diversity than all
implemented internationally by The Reef-World major Indo-Pacific regions. The slight salt in
Foundation and the UN Environment ocean waters is extremely important for the
Programme – which leads to a measurable development of coral polyps.The polyps extract
reduction in the negative environmental calcium from the waters to protect their

e
impacts associated with marine tourism. It skeletons. Hence, mild salinity is a necessity for

n
protects coral reefs through environmentally coral reefs to flourish.

li
friendly guidelines promoting sustainable diving ● Statement 2 is incorrect: Coral reef ecosystems
and snorkelling. It provides the only worldwide have been subject to unprecedented

n
internationally recognised environmental degradation over the past few decades.
o
standards for marine tourism and its robust Disturbances affecting coral reefs include
l.
assessment system measures compliance. At anthropogenic and natural events. Recent
a

least 25% of marine life lives on coral reefs, accelerated coral reef decline seems to be
ri

which are also the centre of marine-related related mostly to anthropogenic impacts
tourism and can account for 40% or more of the (overexploitation, overfishing, increased
te

GDP in some island countries. Reefs are a sedimentation, and nutrient overloading).
particularly fragile ecology, though; the Natural disturbances which cause damage to
a

difference between an increase in the world coral reefs include violent storms, flooding, high
m

temperature of 1.5 or 2.0 C would be fatal to and low-temperature extremes, El Nino


c

them. Coral reefs and other vulnerable marine Southern Oscillation (ENSO) events, etc.
ecosystems may live on in the future due to the Sunscreens and other such cosmetics and
s

Green Fins Hub's increased accessibility to chemicals affect corals and the fragile marine
p

knowledge, best practices, and citizen science. environment.The main chemical culprits are
.u

oxybenzone and octinoxate, which convert


sunburn-causing UV rays into harmless heat on
w

human skin. But once these chemicals are in the


w

Q.99) Ans: C water, they actually decrease corals' defences


Exp: against bleaching, damaging their DNA and
w

hurting their development.

DPP 2023 DAY 181 63


https://upscmaterial.online/
Download From - https://upscmaterial.online/

Contact us :info@onlyias.com

OnlyIAS Nothing Else Visit :dpp.onlyias.in


Contact : +91-7007 931 912

by-national-clean-air-
programme/article65858216.ece

Q.100) Ans: B
Exp:
● Statement 1 is correct: National Clean Air
Programme (NCAP) was launched by the
MoEFCC in January 2019 as a long-term, time-
bound, national level strategy that features:
➢ Making determined efforts to deal with the
air pollution problem across the country in a
comprehensive manner.
➢ Achieving 20% to 30% reduction target in
Particulate Matter concentrations by 2024
where 2017 is kept as the base year for the
comparison of concentration.
➢ Identification of 122 non-attainment cities
across the country based on the 2014-2018
Air Quality data.
➢ Non- Attainment Cities are the cities which do

e
not meet the National Ambient Air Quality

n
Standards.

li
● Statement 2 is incorrect: National Ambient Air

n
Quality Standards are the standards for ambient
air quality set by the Central Pollution Control
o
l.
Board (CPCB). The CPCB has been conferred this
a

power by the Air (Prevention and Control of


Pollution) Act, 1981. Ambient Air Quality
ri

Standards contain 12 pollutants.


te

● Statement 3 is incorrect: The pollutants that are


covered under the National Ambient Air Quality
a

Standards include: Sulphur dioxide (SO2),


m

Nitrogen dioxide (NO2), The particulate matter


having a size less than 10 microns (PM10), The
c

particulate matter having a size less than 2.5


s

microns (PM2.5), Ozone, Lead, Carbon


p

monoxide (CO), Arsenic, Nickel,


.u

Benzene,Ammonia and Benzopyrene.


w

Reference:
https://www.thehindu.com/news/national/cse-
w

analysis-finds-limited-improvement-in-cities-funded-
w

DPP 2023 DAY 181 64


https://upscmaterial.online/

You might also like